You are on page 1of 73

BMO 1965

Problem 1

Sketch f(x) = (x2 + 1)/(x + 1). Find all points where f '(x) = 0 and describe the behaviour when
x or f(x) is large.

Solution

x = -1 and x - 1 are asymptotes. The curve has two branches in the two smaller sectors. The
stationary points are at x = √2 - 1 and x = -√2 - 1.

Problem 2

X, at the centre a circular pond. Y, at the edge, cannot swim, but can run at speed 4v. X can
run faster than 4v and can swim at speed v. Can X escape?

Solution

Answer: yes.

Let 4R be the radius. Let O be the centre of the pond, and C the circle centre O, radius 0.95R.
X swims to C and then swims around C until Y is at the opposite side (with X, O, Y in a
straight line in that order). That is possible because the length of C is less than 1/4 of the
perimeter of the pond. X then heads straight for the edge. X gets there first, because 3.05R <
4πR/4 = 3.14R. X then runs directly away from Y to escape.

Problem 3

Show that np - n is divisible by p for p = 3, 7, 13 and any integer n.

Solution

This is a straightforward piece of bookwork. The result is true for any prime p.
Problem 4

What is the largest power of 10 dividing 100 x 99 x 98 x ... x 1?

Solution

Answer: 24.

There are 20 multiples of 5: 1·5, 2·5, ... , 20·5. Of these 4 are multiples of 25: 1·25, 2·25,
3·25, 4·25. None are multiples of 125. Hence the highest power of 5 dividing 100! is 24. The
highest power of 2 is obviously higher.

Problem 5

Show that n(n + 1)(n + 2)(n + 3) + 1 is a square for n = 1, 2, 3, ... .

Solution

n(n + 1)(n + 2)(n + 3) + 1 = n4 + 6n3 + 11n2 + 6n + 1 = (n2 + 3n + 1)2.

Problem 6

The fractional part of a real is the real less the largest integer not exceeding it. Show that we
can find n such that the fractional part of (2 + √2)n > 0.999 .

Solution

(2 + √2)n + (2 - √2)n is an integer. So the fractional part of (2 + √2)n is 1 - (2 - √2)n. But (2 -


√2) lies between 0 and 1, so (2 - √2)n becomes arbitarily small for large n.

Problem 7

What is the remainder on dividing x + x3 + x9 + x27 + x81 + x243 by x - 1? By x2 - 1?


Solution

Let f(x) = x + x3 + x9 + x27 + x81 + x243. The remainder on dividing by x - 1 is k, satisfying: f(x)
= (x - 1) g(x) + k, for some polynomial g(x). Setting x = 1, k = f(1) = 6. Similarly, the
remainder on dividing by x2 - 1 is ax + b, where f(x) = (x2 - 1) h(x) + ax + b, for some
polynomial h(x). Hence 6 = f(1) = a + b, -6 = f(-1) = -a + b. So a = 6, b = 0, and the remainder
is 6x.

Problem 8

For what real b can we find x satisfying: x2 + bx + 1 = x2 + x + b = 0?

Solution

Answer: b = -2.

If b and x satisfy x2 + bx + 1 = x2 + x + b, then (b - 1)(x - 1) = 0, so either b = 1 or x = 1. If b =


1, then x2 + x + 1 = 0, which has no real roots. If x = 1, then b = -2, which is a solution.

Problem 9

Show that for any real, positive x, y, z, not all equal, we have:

(x + y)(y + z)(z + x) > 8 xyz.

Solution

(x + y)/2 ≥ √(xy) with equality only if x = y.

Problem 10

A chord length √3 divides a circle C radius 1 into two arcs. R is the region bounded by the
chord and the shorter arc. What is the largest area of rectangle than can be drawn in R?

Solution

Answer:
Let the ends of the chord be A and B and its midpoint M. Let O be the centre of the circle.
OM = 1/2 (recognize a 1/2, √3/2, 1 triangle or use Pythagoras). So if we extend OM to meet
the arc at N, them MN = 1/2. Angles AOB and ANB = 120o.

First, we have to show that 2 vertices of a maximal rectangle lie on the chord. Although that is
"obvious", proving it is the hardest part.

If P, Q are vertices of a maximal rectangle on the chord, then they must be equidistant from
M. Otherwise we could enlarge the rectangle by moving the point close to M outwards to until
it was equidistant with the other point.

BMO 1966

Problem 1

Find the greatest and least values of f(x) = (x4 + x2 + 5)/(x2 + 1)2 for real x.

Solution

Answer: 5, 0.95.

f(x) = 5 - (4x4 + 9x2)/(x4 + 2x2 + 1) <= 5, with equality iff x = 0.

Put z = (1 + x2), then f(x) = (z(z - 1) + 5)/z2. Put w = 1/z = 1/(1 + x2), then f(x) = 5w2 - w + 1 =
5(w - 1/10)2 + 95/100 ≥ 0.95 with equality iff w = 1/10 or x = ±3.

[Of course, the question could be done less elegantly but much faster using calculus, but these
problems are aimed at pre-university students, who are assumed not to know calculus.]

Problem

For which distinct, real a, b, c are all the roots of ±√(x - a) ±√(x - b) ±√(x - c) = 0 real?

Solution

Answer: always true.

This is slightly tricky.


Suppose x, a, b, c satisfy the equation for some combination of signs. We show that they must
also satisfy a quadratic, which is independent of the choice of sign.

±√(x - a) = ±√(x - b) ±√(x - c). Squaring: x - a = 2x - b - c ±2 √(x - b)√(x - c). So x + a - b - c


= ±2 √(x - b)√(x - c). Squaring: x2 + 2(a - b - c)x + a2 + b2 + c2 - 2(ab - bc + ca) = 4x2 - 4(b +
c)x + 4bc, so 3x2 - 2(a + b + c)x - (a2 + b2+ c2) + 2(ab + bc + ca) = 0. Set f(x) = 3x2 - 2(a + b +
c)x - (a2 + b2+ c2) + 2(ab + bc + ca). Clearly f(x) > 0 for |x| large. But f(a) = 3a2 - 2a2 - 2ab -
2ca - a2 - b2 - c2 + 2ab + 2bc + 2ca = - (b - c)2 < 0. So f(x) = 0 always has two distinct real
roots, one < a and one > a. The same is true for b and c. So if a > b > c, we have two distinct
real roots, one > a and one < c.

Reading the question strictly, that gives us the answer: if x is a root of the original equations,
then it is real. But that does not tell us whether the original equations have 0, 1 or 2 roots. The
status of the root < c is slightly problematical. It is in fact a real root of the original equation,
but we need complex numbers to see it. For example, let a = 8, b = 5, c = 0. The roots are 9
(giving +3 - 2 - 1 = - 3 + 2 + 1 = 0), which is straightforward, and -1/3 (giving i/√3 (1 + 4 - 5)
= i/√3 (-1 - 4 + 5) = 0). But we still have to prove this is true in general.

Let g(x) = √(x - c) - √(x - b) - √(x - a). For x > a, g(x) is clearly real and continuous. But g(a)
= √(a - c) - √(a - b) > 0. Whereas for x large, g(x) = √x < 0, so g(x) has at least one real root >
a. Similarly, if x < c, then (with a suitable choice of signs) g(x) = i ( √(a - x) - √(b - x) - √(c -
x) ). So g(c) > 0, but g(x) < 0 for x large and negative, so g(x) has at least one real root < c.
But we already know that the equations have at most two real roots, so they have exactly two.
If we choose to regard only values of x >= a as admissible, then they have one real root (and
no imaginary roots).

Problem 3

Sketch y2 = x2(x + 1)/(x - 1). Find all stationary values and describe the behaviour for large x.

Solution

It is not easy to incorporate sketches on this web page, and anyway this is rather trivial
bookwork, so I will just give a brief description.

The obvious point is that x2(x + 1)/(x - 1) is negative for -1 < x < 1, so there are no points on
the graph for these values of x. Also the graph is symmetrical about the x-axis. x = 1 is an
asymptote. Focussing on the positive y-part, y tends to infinity as x tends to 1 from above. It
has a minimum at (1 + √5)/2 and tends to infinity as x2 as x tends to infinity. Obviously, the
reflection in the x-axis has a maximum at the same value of x. Differentiating, the gradient is
infinite at x = -1. So for large negative x, the curve is similar to y = ± x2. It there are two
symmetrically placed inflections and the curve is vertical as it cuts the x-axis.
Problem 4

A1, A2, A3, A4 are consecutive vertices of a regular n-gon. 1/A1A2 = 1/A1A3 + 1/A1A4. What
are the possible values of n?

Solution

Answer: n = 7.

It is a nice question whether one has to show that n = 7 is a solution! [Does the question give
as a fact that there exists a solution?] Clearly, there are no other possible solutions. Suppose
the n-gon has side 1. Then A1A3 and A1A4 are obviously strictly increasing functions of n. So
1/A1A3 + 1/A1A4 is a strictly decreasing function of n, whereas 1/A1A2 is always 1. So there
can be at most one solution. Moreover, for n = 6, the lhs is 1/√3 + 1/2 > 1, whereas for n = 8,
1/√(2 + √2) + (2 - 1) < 1. So if there is a solution it is 7.

If we are not allowed to assume that there is a solution, then we have to prove equality for n =
7.

A kludgy (but easy) proof is as follows. A1A3 = 2 cos π/7, A1A4 = (1 + 2 cos 2π/7). Put c = cos
π/7. The cos 2π/7 = 2c2 - 1. So the result is true if 8c3 - 4c2 - 4c + 1 = 0. Put s = sin π/7. Then
(c + is)7 = - 1. Expanding and comparing real parts, we get: c7 - 21c5(1 - c2) + 35c3(1 - c2)2 -
7c(1 - c2)3 + 1 = 0, or 64c7 - 112c5 + 56c3 - 7c + 1 = 0. But we know that c = -1 is a solution
(not the one we want), so we can factor that out to get: 64c6 - 64c5 - 48c4 + 48c3 + 8c2 - 8c + 1
= 0. But we know that this has three repeated roots (cos π/7 = cos (-π/7) etc), so it should
factorize as the square of a cubic and indeed it does: (8c3 - 4c2 - 4c + 1)2 = 0, so 8c3 - 4c2 - 4c
+ 1 = 0 as required.

Problem 5

A spanner has an enclosed hole which is a regular hexagon side 1. For what values of s can it
turn a square nut side s?

Solution

Answer: 3 - √3 >= s > √(3/2). Note that 3 - √3 = 1.27, √(3/2) = 1.22, so the range is fairly
narrow.

The square must be small enough to fit into the hole and big enough to stick.

The sticking condition is easy. Inscribe a circle in the hexagon. If the square will fit into the
circle, then it obviously does not stick. So we require the diagonal of the square to be > √3, or
s > √(3/2). On the other hand if the diagonal exceeds √3, then it obviously does stick because
the parallel opposite sides of the hexagon are only a distance √3 apart.

It seems fairly clear that the maximum square has sides parallel to a pair of opposite sides and
each vertex a distance x from on endpoint of those sides. For the square to have equal sides,
we then need √3 (1 - x) = 1 + x, or x = 2 - √3, and hence s = 3 - √3. But how do we prove it?

Problem 6

Find the largest interval over which f(x) = √(x - 1) + √(x + 24 - 10√(x - 1) ) is real and
constant.

Solution

Answer: [1, 26].

f(x) is not real for x < 1. For 1 <=x <= 26, we have 0 < √(x - 1) <= 5. Now (5 - √(x - 1) )2 = x
+ 24 - 10√(x - 1). So for 1 <= x <= 26, f(x) = 5. For x > 26, f(x) = 2√(x - 1) - 5.

Problem 7

Prove that √2, √3 and √5 cannot be terms in an arithmetic progression.

Solution

If they are, then for some non-zero rational R, (√5 - √2) = R(√3 - √2). Squaring and
rearranging: √10 = (7 - 5R2)/2 + R2√6. If 7 - 5R2 = 0, then squaring we have an obvious
contradiction: 10 = (49/25) 6. So (7 - 5R2)/2 is non-zero and squaring again gives that √6 is
rational. Contradiction by the standard argument. [If √6 = m/n in lowest terms, then m2 = 6 n2.
2 divides 6, so it must divide m, hence also n. ]

Problem 8

Given 6 different colours, how many ways can we colour a cube so that each face has a
different colour? Show that given 8 different colours, we can colour a regular octahedron in
1680 ways so that each face has a different colour.

Solution
Answer: 30.

Take the colours as C1, C2, ... , C6. We may always rotate the cube so that C1 is on the top
face. There are then 5 choices for the opposite face. The lowest remaining colour may then be
rotated into a fixed position (say north). The three remaining colours may then be distributed
over the three remaining faces in 6 ways. Total 5 x 6 = 30.

Similarly, for the octahedron. Take C1 on the top face. There are 7 possibilities for the bottom
face. There are now 20 (3 chosen from 6) choices for the colours of the three sides adjacent to
the top face. Rotate the lowest colour into a fixed position. There are then 2 choices for the
other two faces. Finally, the remaining 3 colours may be assigned to the remaining 3 faces in
6 ways. Total: 7 x 20 x 2 x 6 = 1680.

Problem 9

The angles of a triangle are A, B, C. Find the smallest possible value of tan A/2 + tan B/2 +
tan C/2 and the largest possible value of tan A/2 tan B/2 tan C/2.

Solution

Answer: equilateral in both cases, √3, 1/(3√3).

Using the familiar tan(a + x) = (tan a + tan x)/(1 - tan a tan x), we have that for a and x in the
interval [0, π/4], tan(a + x) + tan(a - x) =2 tan a (1 + tan2x)/(1 - tan2a tan2x) >= 2 tan a, with
equality iff x = 0. Hence the minimum value of tan A/2 + tan B/2 + tan C/2 occurs when A =
B = C (if any pair of A, B, C are unequal then we can lower the value by replacing the pair by
two angles equal to their mean).

Similarly, we show that for a and x in the interval (0, π/4), tan(a - x) tan(a + x) < tan2a. The
lhs = (tan2a - tan2x)/(1 - tan2a tan2x) = tan2a - tan2x (1 - tan4a)/(1 - tan2a tan2x) < tan2a.

This proves that the maximum value of tan A/2 tan B/2 tan C/2 occurs for A = B = C. For if A
≠ B, then we can increase the value by replacing A and B by their mean (and similarly for any
other unequal pair). This does not deal with the degenerate case of a flat triangle. But if the
angles are A = 2δ, B = 2ε, C = π - (2δ + 2ε), where δ and ε are small, then tan A/2 tan B/2 tan
C/2 = δ ε/(δ + ε) which tends to zero as δ and ε tend to zero. So the flat triangle gives a
smaller value.

Problem 10

One hundred people of different heights are arranged in a 10 x 10 array. X, the shortest of the
10 people who are the tallest in their row, is a different height from Y, the tallest of the 10
people who are the shortest in their column. Is X taller or shorter than Y?
Solution

Answer: taller.

If X and Y are in the same row or column, then the result is immediate (by definition, X is
taller than others in the row, and Y is shorter than others in the column). So suppose they are
not. Take Z in the same row as X and the same column as Y. Then Z is shorter than X and
taller than Y.

Problem 11

(a) Show that given any 52 integers we can always find two whose sum or difference is a
multiple of 100.

(b) Show that given any set 100 integers, we can find a non-empty subset whose sum is a
multiple of 100.

Solution

(a) Consider residues mod 100. We can divide the residues into 51 subsets: two singletons
{0} and {50}, and 49 pairs: {1, 99}, {2, 98}, ... {49, 51}. If we have two nubers from the
same subset, then either their sum or their difference is divisible by 100. But any collection of
52 numbers must have two in the same subset.

(b) Denote the integers as a1, a2, ... , a100. Consider the 100 numbers: a1, a1 + a2, a1 + a2 + a3, ...
, a1 + ... + a100. Either one of them is divisible by 100, in which case we are done, or two of
them must be congruent mod 100 (because there are only 99 non-zero residues mod 100). But
in that case their difference, which is the sum ai + ai+1 + ... + aj for some i ≤ j, is zero mod 100,
as required.

BMO 1967

Problem 1

a, b are the roots of x2 + Ax + 1 = 0, and c, d are the roots of x2 + Bx + 1 = 0. Prove that (a - c)


(b - c)(a + d)(b + d) = B2 - A2.
Solution

We use ab = cd = 1, a + b = -A, c + d = -B repeatedly.

(a - c)(b - c) = 1 + Ac + c2, (a + d)(b + d) = 1 - Ad + d2. So (a - c)(b - c)(a + d)(b + d) = 1 - Ad


+ d2 + Ac - A2 + Ad + c2 - Ac + 1 = 2 + (c2 + d2) - A2 = 2 + (c + d)2 - 2 - A2 = B2 - A2.

Problem 2

Graph x8 + xy + y8 = 0, showing stationary values and behaviour for large values. [Hint: put z
= y/x.]

Solution

Answer: a figure of 8, with the crossing point at the origin, touching both axes at the origin,
extending to x, y = ±0.93, with y = ±x as axes of symmetry.

Problem 3

(a) The triangle ABC has altitudes AP, BQ, CR and AB > BC. Prove that AB + CR >= BC +
AP. When do we have equality?

(b) Prove that if the inscribed and circumscribed circles have the same centre, then the
triangle is equilateral.

Solution

Problem 4

We are given two distinct points A, B and a line l in the plane. Can we find points (in the
plane) equidistant from A, B and l? How do we construct them?

Solution

Provided A does not lie on l, the locus of points equidistant from A and l is (one branch of) a
hyperbola, on the same side of l as A. So if A and B are on opposite sides of l, then there are
no equidistant points.
Assume A and B are on the same side of l. The locus of points equidistant from A and B is the
perpendicular bisector m of the line joining them. This will intersect the other locus in two
points unless m is parallel to l, in which case it only intersects the other locus in one point.

The only remaining possibility is if A (or B) lies on l. In this case the locus of points
equidistant from A and l is the line perpendicular to l at A. So there is just one point
equidistant from A, B and l, unless both A and B lie on l, in which case there are none.

5. Show that (x - sin x)(π - x - sin x) is increasing in the interval (0, π/2). 6. Find all x in [0,
2π] for which 2 cos x ≤ |√(1 + sin 2x) - √(1 - sin 2x)| ≤ √2. Problem 7

Find all reals a, b, c, d such that abc + d = bcd + a = cda + b = dab + c = 2.

Solution

Answer: 1, 1, 1, 1 (one solution); 3, -1, -1, -1 (four solutions).

Either all a, b, c, d are equal, which gives the first solution, or there is an unequal pair.
Suppose a ≠ b. But bcd + a = cda + b, so (a - b)(cd - 1) = 0, and hence cd = 1. Hence a + b = 2.
It follows that ab ≠ 1 (because the only solution to a + b = 2, ab = 1 is a = b = 1). Hence c = d
(since (c - d)(ab - 1) = 0). So either c = d = 1, or c = d = -1. But the first case gives ab = 1, a +
b = 2 (contradiction, since the only possibility then is a = b = 1, whereas we are assuming a ≠
b). So c = d = -1. Hence ab = -3, a + b = 2, so a = 3, b = -1 (or vice versa).

Problem 8

For which positive integers n does 61 divide 5n - 4n?

Solution

Answer: multiples of 3.

Clearly 5 - 4 = 1 (mod 61), 52 - 42 = 9 (mod 61) and 53 - 43 = 0 (mod 61). So it follows that 53m
= 43m (mod 61). In other words 61 does divide 5n - 4n if n is a multiple of 3. It remains to show
that it does not divide 5n - 4n if n = 3m + 1 or 3m + 2.

53m+1 = 5 53m = 5 43m = 43m+1 + 43m (mod 61). But 61 does not divide 43m, and hence it does not
divide 53m+1 - 43m+1. Similarly, 53m+2 = 25 53m = 25 43m = 43m+2 + 9 43m (mod 61). But 61 does not
divide 9 or 43m, so it does not divide 53m+2 - 43m+2.
9. None of the angles in the triangle ABC are zero. Find the greatest and least values of cos2A
+ cos2B + cos2C and the values of A, B, C for which they occurProblem 10

A collects pre-1900 British stamps and foreign stamps. B collects post-1900 British stamps
and foreign special issues. C collects pre-1900 foreign stamps and British special issues. D
collects post-1900 foreign stamps and British special issues. What stamps are collected by (1)
no one, (2) everyone, (3) A and D, but not B?

Solution

Answer: (1) 1900, non-special, British; (2) none; (3) pre-1900, special, British, and post-1900,
non-special, foreign.

Trivial.

Problem 11

The streets for a rectangular grid. B is h blocks north and k blocks east of A. How many
shortest paths are there from A to B?

Solution

Answer: (h + k)!/( h! k! ).

A shortest path must involve h moves north and k moves east, a total of h + k moves. So there
is a (1, 1) correspondence between shortests paths and subsets size h taken from the set 1, 2,
3, ... , (h + k). [Move i is north if i is in the subset and east otherwise.] But there are (h + k) C
h = (h + k)! /( h! k! ) such subsets.

4th BMO 1968

1. C is the circle center the origin and radius 2. Another circle radius 1 touches
C at (2, 0) and then rolls around C. Find equations for the locus of the point P of
the second circle which is initially at (2, 0) and sketch the locus.

2. Cows are put in a field when the grass has reached a fixed height, any cow
eats the same amount of grass a day. The grass continues to grow as the cows
eat it. If 15 cows clear 3 acres in 4 days and 32 cows clear 4 acres in 2 days,
how many cows are needed to clear 6 acres in 3 days?

3. The distance between two points (x, y) and (x', y') is defined as |x - x'| + |y -
y'|. Find the locus of all points with non-negative x and y which are equidistant
from the origin and the point (a, b) where a > b.

4. Two balls radius a and b rest on a table touching each other. What is the
radius of the largest sphere which can pass between them?

5. If reals x, y, z satisfy sin x + sin y + sin z = cos x + cos y + cos z = 0. Show


that they also satisfy sin 2x + sin 2y + sin 2z = cos 2x + cos 2y + cos 2z = 0.

6. Given integers a1, a2, ... , a7 and a permutation of them af(1), af(2), ... , af(7), show
that the product (a1 - af(1))(a2 - af(2)) ... (a7 - af(7)) is always even.

7. How many games are there in a knock-out tournament amongst n people?

8. C is a fixed circle of radius r. L is a variable chord. D is one of the two areas


bounded by C and L. A circle C' of maximal radius is inscribed in D. A is the
area of D outside C'. Show that A is greatest when D is the larger of the two
areas and the length of L is 16πr/(16 + π2).

9. The altitudes of a triangle are 3, 4, 6. What are its sides?

10. The faces of the tetrahedron ABCD are all congruent. The angle between
the edges AB and CD is x. Show that cos x = sin(∠ABC - ∠BAC)/sin(∠ABC +
∠BAC).

11. The sum of the reciprocals of n distinct positive integers is 1. Show that
there is a unique set of such integers for n = 3. Given an example of such a set
for every n > 3.

12. What is the largest number of points that can be placed on a spherical shell
of radius 1 such that the distance between any two points is at least √2? What is
the largest number such that the distance is > √2?

5th BMO 1969


1. Find the condition on the distinct real numbers a, b, c such that (x - a)(x -
b)/(x - c) takes all real values. Sketch a graph where the condition is satisfied
and another where it is not.

2. Find all real solutions to cos x + cos5x + cos 7x = 3.

3. For which positive integers n can we find distinct integers a, b, c, d, a', b', c',
d' greater than 1 such that n2 - 1 = aa' + bb' + cc' + dd'? Give the solution for the
smallest n.

4. Find all integral solutions to a2 - 3ab - a + b = 0.

5. A long corridor has unit width and a right-angle corner. You wish to move a
pipe along the corridor and round the corner. The pipe may have any shape, but
every point must remain in contact with the floor. What is the longest possible
distance between the two ends of the pipe?

6. If a, b, c, d, e are positive integers, show that any divisor of both ae + b and


ce + d also divides ad - bc.

7. (1) f is a real-valued function on the reals, not identically zero, and


differentiable at x = 0. It satisfies f(x) f(y) = f(x+y) for all x, y. Show that f(x) is
differentiable arbitrarily many times for all x and that if f(1) < 1, then f(0) + f(1)
+ f(2) + ... = 1/(1 - f(1) ).

(2) Find the real-valued function f on the reals, not identically zero, and
differentiable at x = 0 which satisfies f(x) f(y) = f(x-y) for all x, y.

8. A square side x has its vertices on the sides of a triangle with inradius r.
Show that 2r > x > r√2.

9. Let An be an n x n array of lattice points (n > 3). Is there a polygon with n2


sides whose vertices are the points of An such that no two sides intersect except
adjacent sides at a vertex? You should prove the result for n = 4 and 5, but
merely state why it is plausible for n > 5.

10. Given a triangle, construct an equilateral triangle with the same area using
ruler and compasses.

6th BMO 1970

. (1) Find 1/log2a + 1/log3a + ... + 1/logna as a quotient of two logs to base 2.
(2) Find the sum of the coefficients of (1 + x - x2)3(1 - 3x + x2)2 and the sum of the
coefficients of its derivative.

2. Sketch the curve x2 + 3xy + 2y2 + 6x + 12y + 4. Where is the center of symmetry?

3. Morley's theorem is as follows. ABC is a triangle. C' is the point of intersection of the
trisector of angle A closer to AB and the trisector of angle B closer to AB. A' and B' are
defined similarly. Then A'B'C' is equilateral. What is the largest possible value of area
A'B'C'/area ABC? Is there a minimum value?

4. Prove that any subset of a set of n positive integers has a non-empty subset whose sum
is divisible by n.

5. What is the minimum number of planes required to divide a cube into at least 300
pieces?

6. y(x) is defined by y' = f(x) in the region |x| ≤ a, where f is an even, continuous function.
Show that (1) y(-a) +y(a) = 2 y(0) and (2) ∫ -aa y(x) dx = 2a y(0). If you integrate
numerically from (-a, 0) using 2N equal steps δ using g(xn+1) = g(xn) + δ x g'(xn), then the
resulting solution does not satisfy (1). Suggest a modified method which ensures that (1)
is satisfied.

7. ABC is a triangle with ∠B = ∠C = 50o. D is a point on BC and E a point on AC such


that ∠BAD = 50o and ∠ABE = 30o. Find ∠BED.

8. 8 light bulbs can each be switched on or off by its own switch. State the total number
of possible states for the 8 bulbs. What is the smallest number of switch changes required
to cycle through all the states and return to the initial state?

9. Find rationals r and s such that √(2√3 - 3) = r1/4 - s1/4.

10. Find "some kind of 'formula' for" the number f(n) of incongruent right-angled
triangles with shortest side n? Show that f(n) is unbounded. Does it tend to infinity

7th BMO 1971

1. Factorise (a + b)7 - a7 - b7. Show that 2n3 + 2n2 + 2n + 1 is never a multiple


of 3.

2. Let a = 99 , b = 9a, c = 9b. Show that the last two digits of b and c are equal.
What are they?

3. A and B are two vertices of a regular 2n-gon. The n longest diameters


subtend angles a1, a2, ... , an and b1, b2, ... , bn at A and B respectively. Show
that tan2a1 + tan2a2 + ... + tan2an = tan2b1 + tan2b2 + ... + tan2bn.

4. Given any n+1 distinct integers all less than 2n+1, show that there must be
one which divides another.

5. The triangle ABC has circumradius R. ∠A ≥ ∠B ≥ ∠C. What is the upper


limit for the radius of circles which intersect all three sides of the triangle?

6. (1) Let I(x) = ∫cx f(x, u) du. Show that I'(x) = f(x, x) + ∫cx ∂f/∂x du.

(2) Find limθ→0 cot θ sin(t sin θ).

(3) Let G(t) = ∫0t cot θ sin(t sin θ) dθ. Prove that G'(π/2) = 2/π.

7. Find the probability that two points chosen at random on a segment of


length h are a distance less than k apart.

8. A is a 3 x 2 real matrix, B is a 2 x 3 real matrix. AB = M where det M = 0


BA = det N where det N is non-zero, and M2 = kM. Find det N in terms of k.

9. A solid spheres is fixed to a table. Another sphere of equal radius is placed


on top of it at rest. The top sphere rolls off. Show that slipping occurs then the
line of centers makes an angle θ to the vertical, where 2 sin θ = μ(17 cos θ -
10). Assume that the top sphere has moment of inertia 2/5 Mr2 about a
diameter, where r is its radius.

8th BMO 1972

1. The relation R is defined on the set X. It has the following two properties: if
aRb and bRc then cRa for distinct elements a, b, c; for distinct elements a, b
either aRb or bRa but not both. What is the largest possible number of
elements in X?

2. Show that there can be at most four lattice points on the hyperbola (x + ay +
c)(x + by + d) = 2, where a, b, c, d are integers. Find necessary and sufficient
conditions for there to be four lattice points.
3. C and C' are two unequal circles which intersect at A and B. P is an
arbitrary point in the plane. What region must P lie in for there to exist a line L
through P which contains chords of C and C' of equal length. Show how to
construct such a line if it exists by considering distances from its point of
intersection with AB or otherwise.

4. P is a point on a curve through A and B such that PA = a, PB = b, AB = c,


and ∠APB = θ. As usual, c2 = a2 + b2 - 2ab cos θ. Show that sin2θ ds2 = da2 +
db2 - 2 da db cos θ, where s is distance along the curve. P moves so that for
time t in the interval T/2 < t < T, PA = h cos(t/T), PB = k sin(t/T). Show that
the speed of P varies as cosec θ.

5. A cube C has four of its vertices on the base and four of its vertices on the
curved surface of a right circular cone R with semi-vertical angle x. Show that
if x is varied the maximum value of vol C/vol R is at sin x = 1/3.

6. Define the sequence an, by a1 = 0, a2 = 1, a3= 2, a4 = 3, and a2n = a2n-5 + 2n,


a2n+1 = a2n + 2n-1. Show that a2n = [17/7 2n-1] - 1, a2n-1 = [12/7 2n-1] - 1.

7. Define sequences of integers by p1 = 2, q1 = 1, r1 = 5, s1= 3, pn+1 = pn2 + 3


qn2, qn+1 = 2 pnqn, rn = pn + 3 qn, sn = pn + qn. Show that pn/qn > √3 > rn/sn and that
pn/qn differs from √3 by less than sn/(2 rnqn2).

8. Three children throw stones at each other every minute. A child who is hit
is out of the game. The surviving player wins. At each throw each child
chooses at random which of his two opponents to aim at. A has probability 3/4
of hitting the child he aims at, B has probability 2/3 and C has probability 1/2.
No one ever hits a child he is not aiming at. What is the probability that A is
eliminated in the first round and C wins.

9. A rocket, free of external forces, accelerates in a straight line. Its mass is M,


the mass of its fuel is m exp(-kt) and its fuel is expelled at velociy v exp(-kt). If
m is small compared to M, show that its terminal velocity is mv/(2M) times its
initial velocity.

9th BMO 1973

1. A variable circle touches two fixed circles at P and Q. Show that the line
PQ passes through one of two fixed points. State a generalisation to ellipses or
conics.

2. Given any nine points in the interior of a unit square, show that we can
choose 3 which form a triangle of area at most 1/8.

3. The curve C is the quarter circle x2 + y2 = r2, x >= 0, y >= 0 and the line
segment x = r, 0 >= y >= -h. C is rotated about the y-axis for form a surface of
revolution which is a hemisphere capping a cylinder. An elastic string is
stretched over the surface between (x, y, z) = (r sin θ, r cos θ, 0) and (-r, -h, 0).
Show that if tan θ > r/h, then the string does not lie in the xy plane. You may
assume spherical triangle formulae such as cos a = cos b cos c + sin b sin c cos
A, or sin A cot B = sin c cot b - cos c cos A.

4. n equilateral triangles side 1 can be fitted together to form a convex


equiangular hexagon. The three smallest possible values of n are 6, 10 or 13.
Find all possible n.

5. Show that there is an infinite set of positive integers of the form 2n - 7 no


two of which have a common factor.

6. The probability that a teacher will answer a random question correctly is p.


The probability that randomly chosen boy in the class will answer correctly is
q and the probability that a randomly chosen girl in the class will answer
correctly is r. The probability that a randomly chosen pupil's answer is the
same as the teacher's answer is 1/2. Find the proportion of boys in the class.

7. From each 10000 live births, tables show that y will still be alive x years
later. y(60) = 4820 and y(80) = 3205, and for some A, B the curve Ax(100-x) +
B/(x-40)2 fits the data well for 60 <= x <= 100. Anyone still alive at 100 is
killed. Find the life expectancy in years to the nearest 0.1 year of someone
aged 70.

8. T: z → (az + b)/(cz + d) is a map. M is the associated matrix


a b

c d

Show that if M is associated with T and M' with T' then the matrix MM' is
associated with the map TT'. Find conditions on a, b, c, d for T4 to be the
identity map, but T2 not to be the identity map.

9. Let L(θ) be the determinant:


x y 1

a + c cos θ b + c sin θ 1

l + n cos θ m + n sin θ 1

Show that the lines are concurrent and find their point of intersection.
10. Write a computer program to print out all positive integers up to 100 of the
form a2 - b2 - c2 where a, b, c are positive integers and a ≥ b + c.

11. (1) A uniform rough cylinder with radius a, mass M, moment of inertia
Ma2/2 about its axis, lies on a rough horizontal table. Another rough cylinder
radius b, mass m, moment of inertia mb2/2 about its axis, rests on top of the
first with its axis parallel. The cylinders start to roll. The plane containing the
axes makes the angle θ with the vertical. Show the forces during the period
when there is no slipping. Write down equations, which will give on
elimination a differential equation for , but you do not need to find the
differential equation.
(2) Such a differential equation is θ2(4 + 2 cos θ - 2 cos2θ + 9k/2) + θ12 sin θ (2
cos θ - 1) = 3g(1 + k) (sin θ /(a + b), where k = M/m. Find θ1 in terms of θ.
Here θ1 denotes dθ/dt and θ2 denotes the second derivative.

10th BMO 1974

1. C is the curve y = 4x2/3 for x ≥ 0 and C' is the curve y = 3x2/8 for x ≥ 0. Find
curve C" which lies between them such that for each point P on C" the area
bounded by C, C" and a horizontal line through P equals the area bounded by
C", C and a vertical line through P.

2. S is the set of all 15 dominoes (m, n) with 1 ≤ m ≤ n ≤ 5. Each domino (m, n)


may be reversed to (n, m). How many ways can S be partitioned into three sets
of 5 dominoes, so that the dominoes in each set can be arranged in a closed
chain: (a, b), (b, c), (c, d), (d, e), (e, a)?

3. Show that there is no convex polyhedron with all faces hexagons.

4. A is the 16 x 16 matrix (ai,j). a1,1 = a2,2 = ... = a16,16 = a16,1 = a16,2 = ... = a16,15 = 1
and all other entries are 1/2. Find A-1.

5. In a standard pack of cards every card is different and there are 13 cards in
each of 4 suits. If the cards are divided randomly between 4 players, so that each
gets 13 cards, what is the probability that each player gets cards of only one
suit?

6. ABC is a triangle. P is equidistant from the lines CA and BC. The feet of the
perpendiculars from P to CA and BC are at X and Y. The perpendicular from P
to the line AB meets the line XY at Z. Show that the line CZ passes through the
midpoint of AB.
7. b and c are non-zero. x3 = bx + c has real roots α, β, γ. Find a condition which
ensures that there are real p, q, r such that β = pα2 + qα + r, γ = pβ2 qβ+ r, α = pγ2
+ qγ + r.

8. p is an odd prime. The product (x + 1)(x + 2) ... (x + p - 1) is expanded to


give ap-1xp-1 + ... + a1x + a0. Show that ap-1 = 1, ap-2 = p(p-1)/2!, 2ap-3 = p(p-1)(p-
2)/3! + ap-2(p-1)(p-2)/2!, ... , (p-2)a1 = p + ap-2(p-1) + ap-3(p-2) + ... + 3a2, (p-1)a0 =
1 + ap-2 + ... + a1. Show that a1, a2, ... , ap-2 are divisible by p and (a0 + 1) is
divisible by p. Show that for any integer x, (x+1)(x+2) ... (x+p-1) - xp-1 + 1 is
divisible by p. Deduce Wilson's theorem that p divides (p-1)! + 1 and Fermat's
theorem that p divides xp-1 - 1 for x not a multiple of p.

9. A uniform rod is attached by a frictionless joint to a horizontal table. At time


zero it is almost vertical and starts to fall. How long does it take to reach the
table? You may assume that ∫ cosec x dx = log |tan x/2|.

10. A long solid right circular cone has uniform density, semi-vertical angle x
and vertex V. All points except those whose distance from V lie in the range a to
b are removed. The resulting solid has mass M. Show that the gravitational
attraction of the solid on a point of unit mass at V is 3/2 GM(1 + cos x)/(a2 + ab
+ b2).

11th BMO 1975

1. Find all positive integer solutions to [11/3] + [21/3] + ... + [(n3 - 1)1/3] = 400.

2. The first k primes are divided into two groups. n is the product of the first
group and n is the product of the second group. M is any positive integer
divisible only by primes in the first group and N is any positive integer
divisible only by primes in the second group. If d > 1 divides Mm - Nn, show
that d exceeds the kth prime.

3. Show that if a disk radius 1 contains 7 points such that the distance between
any two is at least 1, then one of the points must be at the center of the disk.
[You may wish to use the pigeonhole principle.]

4. ABC is a triangle. Parallel lines are drawn through A, B, C meeting the


lines BC, CA, AB at D, E, F respectively. Collinear points P, Q, R are taken on
the segments AD, BE, CF respectively such that AP/PD = BQ/CE = CR/RF =
k. Find k.

5. Let nCr represent the binomial coefficient n!/( (n-r)! r! ). Define f(x) =
(2m)C0 + (2m)C1 cos x + (2m)C2 cos 2x + (2m)C3 cos 3x + ... + (2m)C(2m)
cos 2mx. Let g(x) = (2m)C0 + (2m)C2 cos 2x + (2m)C4 cos 4x + ... +
(2m)C(2m) cos 2mx. Find all x such that x/π is irrational and limm→∞ g(x)/f(x)
= 1/2. You may use the identity: f(x) = (2 cos(x/2) )2m cos mx.

6. Show that for n > 1 and real numbers x > y > 1, (xn+1 - 1)/(xn - x) > (yn+1 -
1)/(yn - y).

7. Show that for each n > 0 there is a unique set of real numbers x1, x2, ... , xn
such that (1 - x1)2 + (x1 - x2)2 + ... + (xn-1 - xn)2 + xn2 = 1/(n + 1).

8. A wine glass has the shape of a right circular cone. It is partially filled with
water so that when tilted the water just touches the lip at one end and extends
halfway up at the other end. What proportion of the glass is filled with water?

12th BMO 1976

1. ABC is a triangle area k. Let d be the length of the shortest line segment
which bisects the area of the triangle. Find d. Give an example of a curve
which bisects the area and has length < d.

2. Prove that x/(y + z) + y/(z + x) + z/(x + y) ≥ 3/2 for any positive reals x, y,
z.

3. Given 50 distinct subsets of a finite set X, each containing more than | X


|/2 elements, show that there is a subset of X with 5 elements which has at
least one element in common with each of the 50 subsets.

4. Show that 8n19 + 17 is not prime for any non-negative integer n.

5. aCb represents the binomial coefficient a!/( (a - b)! b! ). Show that for n a
positive integer, r ≤ n and odd, r' = (r - 1)/2 and x, y reals we have: ∑0r' nC(r-i)
nCi (xr-iyi + xiyr-i) = ∑0r' nC(r-i) (r-i)Ci xiyi(x + y)r-2i.

6. A sphere has center O and radius r. A plane p, a distance r/2 from O,


intersects the sphere in a circle C center O'. The part of the sphere on the
opposite side of p to O is removed. V lies on the ray OO' a distance 2r from
O'. A cone has vertex V and base C, so with the remaining part of the sphere it
forms a surface S. XY is a diameter of C. Q is a point of the sphere in the
plane through V, X and Y and in the plane through O parallel to p. P is a point
on VY such that the shortest path from P to Q along the surface S cuts C at 45
deg. Show that VP = r√3 / √(1 + 1/√5).
Problem 2

Prove that x/(y + z) + y/(z + x) + z/(x + y) ≥ 3/2 for any positive reals x, y, z.

Solution

If x + y + z = k, then (1/(k-x) + 1/(k-y) + 1/(k-z) )(k-x + k-y + k-z) ≥ 9 by Cauchy Schwartz.


Hence 1/(y+z) + 1/(z+x) + 1/(x+y) ≥ 9/(2k). So k/(y+z) + k/(z+x) + k/(x+y) ≥ 9/2. Subtracting
(y+z)/(y+z) + (z+x)/(z+x) + (x+y)/(x+y) = 3 gives the result.

13th BMO 1977

1. f(n) is a function on the positive integers with non-negative integer values


such that: (1) f(mn) = f(m) + f(n) for all m, n; (2) f(n) = 0 if the last digit of n is
3; (3) f(10) = 0. Show that f(n) = 0 for all n.

2. S is either the incircle or one of the excircles of the triangle ABC. It touches
the line BC at X. M is the midpoint of BC and N is the midpoint of AX. Show
that the center of S lies on the line MN.

3. (1) Show that x(x - y)(x - z) + y(y - z)(y - x) + z(z - x)(z - y) ≥ 0 for any
non-negative reals x, y, z.
(2) Hence or otherwise show that x6 + y6 + z6 + 3x2y2z2 ≥ 2(y3z3 + z3x3 + x3y3)
for all real x, y, z.

4. x3 + qx + r = 0, where r is non-zero, has roots u, v, w. Find the roots of r2x3


+ q3x + q3 = 0 (*) in terms of u, v, w. Show that if u, v, w are all real, then (*)
has no real root x satisfying -1 < x < 3.

5. Five spheres radius a all touch externally two spheres S and S' of radius a.
We can find five points, one on each of the first five spheres, which form the
vertices of a regular pentagon side 2a. Do the spheres S and S' intersect?

6. Find all n > 1 for which we can write 26(x + x2 + x3 + ... + xn) as a sum of
polynomials of degree n, each of which has coefficients which are a
permutation of 1, 2, 3, ... , n.
14th BMO 1978

1. Find the point inside a triangle which has the largest product of the distances
to the three sides.

2. Show that there is no rational number m/n with 0 < m < n < 101 whose
decimal expansion has the consecutive digits 1, 6, 7 (in that order).

3. Show that there is a unique sequence a1, a2, a3, ... such that a1 = 1, a2 > 1,
an+1an-1 = an3 + 1, and all terms are integral.

4. An altitude of a tetrahedron is a perpendicular from a vertex to the opposite


face. Show that the four altitudes are concurrent iff each pair of opposite edges
is perpendicular.

5. There are 11000 points inside a cube side 15. Show that there is a sphere
radius 1 which contains at least 6 of the points.

6. Show that 2 cos nx is a polynomial of degree n in (2 cos x). Hence or


otherwise show that if k is rational then cos kπ is 0, ±1/2, ±1 or irrational.

15th BMO 1979

1. Find all triangles ABC such that AB + AC = 2 and AD + BD = √5, where


AD is the altitude.

2. Three rays in space have endpoints at O. The angles between the pairs are α,
β, γ, where 0 < α < β < γ. Show that there are unique points A, B, C, one on
each ray, so that the triangles OAB, OBC, OCA all have perimeter 2s. Find
their distances from O.

3. Show that the sum of any n distinct positive odd integers whose pairs all
have different differences is at least n(n2 + 2)/3.

4. f(x) is defined on the rationals and takes rational values. f(x + f(y) ) = f(x)
f(y) for all x, y. Show that f must be constant.
5. Let p(n) be the number of partitions of n. For example, p(4) = 5: 1 + 1 + 1 +
1, 1 + 1 + 2, 2 + 2, 1 + 3, 4. Show that p(n+1) ≥ 2p(n) - p(n-1).

6. Show that the number 1 + 104 + 108 + ... + 104n is not prime for n > 0.

16th BMO 1980

1. Show that there are no solutions to an + bn = cn, with n > 1 is an integer, and
a, b, c are positive integers with a and b not exceeding n.

2. Find a set of seven consecutive positive integers and a polynomial p(x) of


degree 5 with integer coefficients such that p(n) = n for five numbers n in the
set including the smallest and largest, and p(n) = 0 for another number in the
set.

3. AB is a diameter of a circle. P, Q are points on the diameter and R, S are


points on the same arc AB such that PQRS is a square. C is a point on the same
arc such that the triangle ABC has the same area as the square. Show that the
incenter I of the triangle ABC lies on one of the sides of the square and on the
line joining A or B to R or S.

4. Find all real a0 such that the sequence a0, a1, a2, ... defined by an+1 = 2n - 3an
has an+1 > an for all n ≥ 0.

5. A graph has 10 points and no triangles. Show that there are 4 points with no
edges between them.

17th BMO 1981

Problem 1

ABC is a triangle. Three lines divide the triangle into four triangles and three pentagons. One
of the triangle has its three sides along the new lines, the others each have just two sides along
the new lines. If all four triangles are congruent, find the area of each in terms of the area of
ABC.

Solution
The obvious configuration has ∠EDF = ∠EJI, so KH is parallel to AC and similarly LI
parallel to BC, and GJ parallel to AB. Then to get equal area we need E,F to trisect LI and
similarly for the other two lines. Suppose GH = kBC. Then GJ = 3kAB, so CG = 3kBC.
Hence CH = 2kBC and similarly BG = 2kBC, so k = 1/5 and the area of each small triangle is
1/25 area ABC.

However, there are other configurations. For example, suppose we take GJ parallel to AB and
KH parallel to AC, but LI not parallel to BC. Take GH = a, DH = b, GD = c. Then GDH is
evidently similar to BAC because of the parallel lines. We find DE = b, EJ = b, so GJ = 2b+c.
GJC is similar to GDH, so GC = a(2b+c)/c. Similarly, DF = KF = c, so KH = b+2c. Hence BH
= a(b+2c)/b. Hence BC = a(bc+2b2+2c2)/bc. So BC/GH = (bc+2b2+2c2)/bc and hence the area
of a small triangle is 1/(1 + 2b/c + 2c/b)2 times the area of ABC. Here the ratio can take any
value less than 1/25 depending on the ratio b/c.

Problem 2

An axis of a solid is a straight line joining two points on its boundary such that a rotation
about the line through an angle greater than 0 deg and less than 360 deg brings the solid into
coincidence with itself. How many such axes does a cube have? For each axis indicate the
minimum angle of rotation and how the vertices are permuted.

Answer

3 through the centers of two opposite faces, minimum angle 90o


6 through the midpoints of two opposite edges, minimum angle 180o
4 through opposite vertices, minimum angle 120o
Problem 3

Find all real solutions to x2y2 + x2z2 = axyz, y2z2 + y2x2 = bxyz, z2x2 + z2y2 = cxyz, where a, b,
c are fixed reals.

Answer

(x,y,z) = (t,0,0), (0,t,0), (0,0,t) are solutions for any t and any a,b,c.
If a, b, c are the sides of a triangle (so a,b,c > 0 and a+b > c etc), or -a, -b, -c are the sides of a
triangle, then x = ±√((s-b)(s-c)), y = ±√((s-a)(s-c)), z = ±√((s-a)(s-b)), where we have either 0
or 2 minus signs, are solutions.

Solution

If x = 0, then y2z2 = 0, so y or z = 0. If x = y = 0, then z can be arbitrary. Similarly for the


other pairs. Thus we get solutions (x,y,z) = (0,0,t), (0,t,0), (t,0,0) for any t. So suppose xyz ≠
0.

Subtracting the second equation from the first, x2z2-y2z2 = (a-b)xyz. Adding to the last
equation, x2z2 = (a-b+c)xyz/2. Similarly y2z2 = (-a+b+c)xyz/2. Hence x2y2z4 = (a-b+c)(-
a+b+c)x2y2z2/4, so z2 = (a-b+c)(-a+b+c)/4 = (s-a)(s-b), where s = (a+b+c)/2. Similarly, x2 = (s-
b)(s-c), y2 = (s-a)(s-c). However, this solution is only possible if a, b, c satisfy certain
conditions.

If s-a > 0, then z2 > 0 implies s-b > 0 and x2 > 0 implies s-c > 0. Adding s-a > 0 and s-b > 0
gives c > 0. Similarly, a > 0 and b > 0. So a, b, c must be the sides of a triangle. Similarly, if s-
a < 0, then s-b < 0 and s-c < 0, and hence a, b, c < 0. In this case -a, -b, -c are the sides of a
triangle.

Problem 4

Find the remainder on dividing x81 + x49 + x25 + x9 + x by x3 - x.

Solution

Put p(x) = x81 + x49 + x25 + x9 + x = q(x)(x3 - x) + ax2 + bx + c. Putting x = 0 gives c = 0.


Putting x = 1 gives 5 = a + b, putting x = -1 gives -5 = a - b, so a = 0, b = 5. Hence the
remainder is 5x.

Problem 5
The sequence u0, u1, u2, ... is defined by u0 = 2, u1 = 5, un+1un-1 - un2 = 6n-1. Show that all terms
of the sequence are integral.

Solution

We show by induction that un = 2n + 3n. True for n = 0 and 1. Suppose it is true for n and n-1,
then un+1(2n-1+3n-1) - (2n+3n)2 = 6n-1. Hence un+1(2n-1+3n-1) = 22n + 32n + 13·6n-1 = (2n+1+3n+1)(2n-
1
+3n-1). Hence un+1 = 2n+1+3n+1, so the result is true for n+1 and hence for all n.

Problem 6

Show that for rational c, the equation x3 - 3cx2 - 3x + c = 0 has at most one rational root.

Solution

Suppose it has a rational root k. Then c = (k3-3k)/(3k2-1), so the equation is (3k2-1)x3 - 3(k3-
3k)x2 - 3(3k2-1)x + k3-3k = 0, which factorises as (x-k)( (3k2-1)x2 + 8kx - (k2-3) ) = 0. The
roots of the quadratic are -4k/(3k2-1) ± (√3)(k2+1)/(3k2-1), which are irrational.

Problem 7

If x and y are non-negative integers, show that there are non-negative integers a, b, c, d such
that x = a + 2b + 3c + 7d, y = b + 2c + 5d iff 5x ≥ 7y.

Solution

If 5x = 7y, then y = 5d for some non-negative integer d, and we can take a = b = c = 0. If 5x =


7y+1, then x = 7d+3 for some non-negative integer d, so y = 5d+2, c = 1, a = b = 0. If 5x =
7y+2, then x = 7d+6, so y = 5d+4, c = 2, a = b = 0. If 5x = 7y+3, then x = 7d+2, y = 5d+1, b =
1, a = c = 0. If 5x = 7y+4, then x = 7d+5, y = 5d+3, a = 0, b = c = 1. If 5x = 7y + n, for n > 4,
then we can take m as the residue of n mod 5, a = n-m and b,c,d as above.

The implication the other way is trivial.

18th BMO 1982


Problem 1

ABC is a triangle. The angle bisectors at A, B, C meet the circumcircle again at P, Q , R


respectively. Show that AP + BQ + CR > AB + BC + CA.

Solution

18th BMO 1982

Problem 1

ABC is a triangle. The angle bisectors at A, B, C meet the circumcircle again at P, Q , R


respectively. Show that AP + BQ + CR > AB + BC + CA.

Solution

See Australian 82/3

Problem 2

The sequence p1, p2, p3, ... is defined as follows. p1 = 2. pn+1 is the largest prime divisor of
p1p2 ... pn + 1. Show that 5 does not occur in the sequence.

Solution

See Australian 82/5

Problem 3

a is a fixed odd positive integer. Find the largest positive integer n for which there are no
positive integers x, y, z such that ax + (a + 1)y + (a + 2)z = n.
Answer

(a2+5a+4)/2

Solution

Put s = x+y+z, so ax + (a+1)y + (a+2)z = as+y+2z. For fixed s, the smallest value this can
achieve is as+3 and the largest is (a+2)s-3. It can also take all values in between, because
(x,y,z) = (s-2-m,m+1,1) gives as+3+m for 1 ≤ m ≤ s-3 and (x,y,z) = (1,s-2-m,m+1) gives
as+s+m for 1 ≤ m ≤ s-3.

We get a gap between the values for s and the values for s+1 iff (a+2)s-3 < a(s+1)+2 or 2s <
a+5. We are given that a is odd, so we have a gap iff s < (a+5)/2. The largest element in the
last gap is thus a(a+5)/2 + 2 = (a2+5a+4)/2.

Problem 4

a and b are positive reals and n > 1 is an integer. P1 (x1, y1) and P2 (x2, y2) are two points on
the curve xn - ayn = b with positive real coordinates. If y1 < y2 and A is the area of the triangle
OP1P2, show that by2 > 2ny1n-1a1-1/nA.

Solution

The graph shows a curve for n = 2 and a curve for n = 3. It is easy to check that A = (x1y2-
x2y1)/2.

Now x/y = (a + b/yn)1/n = a1/n(1 + b/(ayn))1/n. Now for any positive t we have (1 + t)1/n < 1 + t/n
(eg take nth powers and expand rhs by binomial theorem). Hence x/y < a1/n(1 + b/(anyn)). Also
x/y > a1/n. Hence x1/y1 - x2/y2 < a1/nb/(ny1na). So x1y2 - x2y1 < by2/(ny1n-1a1-1/n). So by2 > 2ny1n-
1 1-/n
a A.

Problem 5

p(x) is a real polynomial such that p(2x) = 2k-1(p(x) + p(x + 1/2) ), where k is a non-negative
integer. Show that p(3x) = 3k-1(p(x) + p(x + 1/3) + p(x + 2/3) ).

Solution

p(2x+1) = 2k-1(p(x + 1/2) + p(x+1)), so p(2x+1) - p(x) = 2k-1(p(x+1) - p(x)). Put q(x) = p(x+1) -
p(x), then q(2x) = 2k-1q(x). Put q(x) = ∑ aixi. Then 2iai = 2k-1ai, so ai = 0 except for i = k-1. If k
= 0, then q(x) = 0, so p(x+1) = p(x). Hence p(x) = b for some constant b, so p(3x) = b = 1/3 (b
+ b + b) = 3k-1(p(x) + p(x + 1/3) + p(x + 2/3) ). So assume k > 0. Hence q(x) = axk-1.

Now put r(x) = p(3x) - 3k-1(p(x) + p(x + 1/3) + p(x + 2/3) ). Then r(x + 1/3) - r(x) = q(3x) - 3k-
1
q(x) = a3k-1xk-1 - 3k-1axk-1 = 0. Hence r(x) is a constant. Put r(x) = c.

We have p(6x) = p(2·3x) = 2k-1(p(3x) + p(3(x+1/6)) ) = 6k-1(p(x) + p(x + 1/6) + p(x + 1/3) +
p(x + 1/2) + p(x + 2/3) + p(x + 5/6) ) + 2kc. But also p(6x) = p(3·2x) = 3k-1(p(2x) +
p(2(x+1/6)) + p(2(x+1/3)) ) + c = 6k-1(p(x) + p(x + 1/6) + p(x + 1/3) + p(x + 1/2) + p(x + 2/3)
+ p(x + 5/6) ) + c. So 2kc = c. Hence c = 0.

19th BMO 1983

Problem 1

Given points A and B and a line l, find the point P which minimises PA2 + PB2 + PN2, where
N is the foot of the perpendicular from P to l. State without proof a generalisation to three
points.

Solution
Let M be the midpoint of AB. We have the familiar result that PA2 + PB2 = 2PM2 + 2MA2.
Hence P must minimise 2PM2 + PN2 and hence lie on MC, the perpendicular from M to the
line. Let G be the point on the segment MC such that GC = 2MG. Suppose P is a distance x
from G, taking positive distances towards C. Then PC2 + 2PM2 = (GC-x)2 + 2(GM+x)2 = GC2
+ 2GM2 + 2x(2GM-GC) + 2x2 = GC2 + 2GM2 + 2x2, which is minimised by taking x = 0. So
we should take P at G.

For 3 points and a line, take D to be the centroid of the three points and D' to be the foot of the
perpendicular from D to the line. Then P should be taken at the centroid of the three points
and D'.

Problem 2

Each pair of excircles of the triangle ABC has a common tangent which does not contain a
side of the triangle. Show that one such tangent is perpendicular to OA, where O is the
circumcenter of ABC.

Solution

Let the common tangent meet the lines AC, AB at B', C' respectively, and let the line AO
meet the line B'C' at Z. Then B' and C' are the reflections of B, C in the line EF, so ∠AB'C' =
∠B. But ∠ZAB' = ∠OAC = 90o - ∠B (because ∠AOC = 2∠B). Hence ∠AZB' = 90o.

Problem 3

l, m, and n are three lines in space such that neither l nor m is perpendicular to n. Variable
points P on l and Q on m are such that PQ is perpendicular to n, The plane through P
perpendicular to m meets n at R, and the plane through Q perpendicular to l meets n at S.
Show that RS has constant length.

Solution
Use vectors. Take any origin O. Take vectors λ, μ, ν parallel to l, m, n. Take fixed points A, B,
C on l, m, n. Write the vectors OA, OB, OC as a, b, c. Then OP is a + hλ, OQ is b + kμ. Since
PQ is perpendicular to n, we have (a - b + hλ - kμ).ν = 0 (*). Take R to have vector c + rν.
Since PR is perpendicular to m, we have (c - a + rν - hλ).μ = 0, so (ν.μ)r = a.μ + hλ.μ - c.μ.
Similarly, take S to have vector c + sν. Since QS is perpendicular to l, we have (c - b + sν -
kμ).λ = 0, so (ν.λ)s = b.λ + kμ.λ - c.λ. Hence (ν.μ)(ν.λ)(r - s) = (ν.μ)(c-b).λ + (&lambda.μ)(b-
a).ν + (&nu.λ)(a-c).μ, where we have used (*) to combine the h and k terms. Thus the
distance RS is independent of h, k.

Problem 4

Show that for any positive reals a, b, c, d, e, f we have ab/(a + b) + cd/(c + d) + ef/(e + f) ≤ (a
+ c + e)(b + d + f)/(a + b + c + d + e + f).

Solution

With some work (28 terms cancel) multiplying across reduces ab/(a+b) + cd/(c+d) ≤ (a+c)
(b+d)/(a+b+c+d) to (ad-bc)2 ≥ 0, which is obviously true. The trick is now to apply this again:
(a+c)(b+d)/(a+b+c+d) + ef/(e+f) ≤ (a+c+e)(b+d+f)/(a+b+c+d+e+f).

Problem 5

How many permutations a, b, c, d, e, f, g, h of 1, 2, 3, 4, 5, 6, 7, 8 satisfy a < b, b > c, c < d,


d> e, e < f, f > g, g < h?

Answer

1385

Solution

We use the principle of inclusion and exclusion to simplify the counting. Let S be the set of
permutations with a < b, c < d, e < f, g < h. Evidently S has 8!/24 = 2520 members. Let A1 be
the subset with b < c, A2 the subset with d < e, A3 the subset with f < g. Let A12 be the subset
with a < b and d < e, A13 the subset with a < b and f < g, A23 the subset with d < e and f < g,
and A123 the subset with a < b and d < e and f < g.
Members of A1 satisfy a < b < c < d and e < f and g < h. We can choose a,b,c,d in 8C4 ways,
then d,e in 6 ways. That determines the permutation, so A1 has 420 members. Similarly, A2
and A3 have 420 members. Members of A12 satisfy a < b < c < d < e < f and g < h. We can
choose g and h in 8C2 = 28 ways and the permutation is then determined, so A12 has 28
members. Similarly, A23. Members of A13 satisfy a < b < c < d and e < f < g < h. We can
choose a, b, c, d in 8C4 = 70 ways and the permutation is then determined, so A13 has 70
members. A123 obviously has just one member. Hence the number of elements satisfying the
conditions in the question is |S| - |A1| - |A2| - |A3| + |A12| + |A13| + |A23| - |A123| = 2520 - 3·420 +
2·28 + 70 - 1 = 1385.

Problem 6

Find all positive integer solutions to (n + 1)m = n! + 1.

Answer

(n,m) = (1,1), (2,1), (4,2)

Solution

If n = 1, then m must be 1, which gives a solution. So assume n > 1. Hence n! is even and n! +
1 is odd, so n must be even. If n = 2, then m must be 1 and we have a solution. If n = 4, then
5m = 25, so m must be 2 which is a solution. So assume n > 4. Then 2, n/2 and n are separate
terms of n!, so n2 divides n! and hence also (n+1)m - 1. Expanding by the binomial theorem,
we see that n must divide m. Hence, in particular m ≥ n. But now (n+1)m ≥ (n+1)n > n! + 1. So
there are no solutions for n > 4.

Problem 7

Show that in a colony of mn + 1 mice, either there is a set of m + 1 mice, none of which is a
parent of another, or there is an ordered set of n + 1 mice (M0, M1, M2, ... , Mn) such that Mi is
the parent of Mi-1 for i = 1, 2, ... , n.

Solution

This is just a variant of the well-known Erdös-Szekeres result. For each mouse mi let ti be the
length of the longest parent sequence starting at mi, where n1, n2, ... is a parent sequence if nj is
the parent of nj-1 for each j. If any ti ≥ n+1, then we are done. So assume all ti ≤ n. So there
must be at least m+1 mice with the same ti. Now suppose one of these M was the parent of
another M'. Then we could get a parent sequence of length ti+1 for M' by taking M' followed
by M and its parent sequence. That contradicts the maximality of ti. So none of the m+1 mice
is the parent of another.

20th BMO 1984

Problem 1

In the triangle ABC, ∠C = 90o. Find all points D such that AD·BC = AC·BD = AB·CD/√2.

Solution

Take rectangular coordinates with C as (0,0), A as (a,0), B as (0,b). Take D as (x,y). Then ((x-
a)2+y2)b2 = a2(x2+(y-b)2) = (a2+b2)(x2+y2)/2.

Hence (a2-b2)(x2+y2) + 2ab(bx-ay) = 0 and (a2-b2)(x2+y2) - 4a2by + 2a2b2 = 0. Hence bx + ay =


ab. So D lies on the line AB. Substituting back, we find (x,y) = (ab/(a+b),ab/(a+b)) or (ab/(a-
b),ab/(a-b)).

Geometrically, the two points are the intersections of the internal and external angle bisectors
of angle C with the line AB.

Problem 2

ABCD is a tetrahedron such that DA = DB = DC = d and AB = BC = CA = e. M and N are


the midpoints of AB and CD. A variable plane through MN meets AD at P and BC at Q.
Show that AP/AD = BQ/BC. Find the value of this ratio in terms of d and e which minimises
the area of MQNP.

Answer

(d2+e2)/4d2

Solution
Call the plane Π. If Π is parallel to BD, then P must be the midpoint of AD, and Q must be the
midpoint of BC, so AP/AD = BQ/BC = 1/2. So assume it is not parallel and let the point of
intersection be X. The line PM lies in the plane ABD and in Π so it must be their intersection.
But X must lie in the intersection of ABD and Π, so it must lie on PM. Similarly it must lie on
QN. Now applying Menelaus to the line XNQ and triangle BCD we get (BQ/QC)(CN/ND)
(DX/XB) = 1. Applying it to line MPX and triangle ABD we get (AP/PD)(DX/XB)(BM/MA)
= 1. Hence AP/PD = BQ/QC and hence AP/AD = BQ/BC.

It follows that the midpoint of PQ must lie on MN. That is obvious if we use vectors, because
P = λA + (1-λ)D, Q = λB + (1-λ)C, so ½P + ½Q = λM + (1-λ)N. Hence area PNM = area
QNM and it is sufficient to minimise area PNM.

Take G to be the midpoint of MN and take P to be the foot of the perpendicular from G to
AD. Then G is the centroid of ABCD. Since ABC is equilateral and DA = DB = DC, the
plane ADG is perpendicular to BC. Hence GP is perpendicular to BC. Let L be the midpoint
of AC, then LM is parallel to BC and hence perpendicular to GP. Also NL is parallel to AD
which is perpendicular to GP, so NL is perpendicular to GP. So the plane LMN is
perpendicular to GP and hence MN is perpendicular to GP. So GP is the shortest distance
between the lines AD and MN and hence this choice of P minimises area MNP.

The line DG meets the plane ABC at the center O of the triangle ABC. ∠AOG = ∠APG =
90o, so AOGP is cyclic and hence DP·DA = DG·DO = (3/4)DO2 = (3/4)(AD2 - AO2) = (3/4)
(d2 - e2/3). Hence DP = (3d2-e2)/(4d) and AP = (d2+e2)/4d, so the required ratio AP/AD =
(d2+e2)/4d2.
Problem 3

Find the maximum and minimum values of cos x + cos y + cos z, where x, y, z are non-
negative reals with sum 4π/3.

Answer

max 3/2 at 0,0,4π/3


min 0 at 0,2π/3,2π/3

Solution

wlog x ≤ y ≤ z. So 4π/9 ≤ z ≤ 4π/3. Put z = 4π/3 - k, so 0 ≤ k ≤ 8π/9. Then cos x + cos y + cos
z = cos(4π/3 - k) + 2 cos k/2 cos(y-x)/2. Since cos(k/2) ≥ 0, the expression is maximised by
taking x = y so that cos(y-x)/2 = 1. That gives cos x + cos y + cos z = cos(4π/3 - k) + 2 cos k/2
= -1/2 cos k - (√3)/2 sin k + 2 cos k/2 = - cos2k/2 + 1/2 + 2 cos k/2 - (√3)/2 sin k = 3/2 - (cos
k/2 - 1)2 - (√3)/2 sin k. Since k < π, sin k ≥ 0, so we maximise this expression by taking k = 0
which gives (cos k/2 - 1) = 0 and sin k = 0.

We minimise cos(4π/3 - k) + 2 cos k/2 cos(y-x)/2 by taking cos(y-x)/2 as small as possible, in


other words by taking x = 0, y = k, giving cos x + cos y + cos z = cos(4π/3 - k) + 2 cos2k/2 =
cos(4π/3 - k) + cos k + 1 = 1 + 2 cos 2π/3 cos(2π/3 - k) = 1 - cos(2π/3 - k), which is minimised
by taking k = 2π/3 to give 0.

Problem 4

Let bn be the number of partitions of n into non-negative powers of 2. For example b4 = 4: 1 +


1 + 1 + 1, 1 + 1 + 2, 2 + 2, 4. Let cn be the number of partitions which include at least one of
every power of 2 from 1 up to the highest in the partition. For example, c4 = 2: 1 + 1 + 1 + 1,
1 + 1 + 2. Show that bn+1 = 2cn.

Solution

Let Cn be the set of partitions of n into non-negative powers of 2, where there is at least one of
every power of 2 from 1 up to the highest in the partition, so |Cn| = cn. Let Dn be the set of
partitions of n into non-negative powers of 2, where there is just one of the highest power of 2
(for example 4 = 4 belongs to D4, but 4 = 2 + 2 does not). Let En be the set of partitions of n
into non-negative power of 2, where there is more than one of the highest power of 2.
Obviously bn+1 = |Dn+1| + |En+1|. We show that |Dn+1| = |En+1| = cn.
Given a partition n = ∑0k ai2i in Cn, we see that n+1 = ∑0k (ai-1)2i + 2k+1 is a partition in Dn+1.
Moreover this is obviously a bijection between the two sets. Hence |Dn+1| = |Cn|. Similarly,
given a partition n = ∑0k ai2i in Cn, we see that n+1 = ∑0k-1 (ai-1)2i + (ak+1)2k is a partition in
En+1, and again this is obviously a bijection between the two sets. Hence |En+1| = |Cn|.

Problem 5

Show that for any positive integers m, n we can find a polynomial p(x) with integer
coefficients such that | p(x) - m/n | ≤ 1/n2 for all x in some interval of length 1/n.

Solution

Take p(x) = (m/n) (nx - 1)2k+1 + m/n. Exanding by the binomial theorem, it is obvious that p(x)
has integer coefficients. Take the interval to be 1/n ± 1/2n. Then we have |p(x) - m/n| ≤ (m/n)
1/22k+1, which can be made arbitrarily small (and certainly < 1/n2) for sufficiently large k.

23rd BMO 1987 missing!


24th BMO 1988

Problem 1

ABC is an equilateral triangle. S is the circle diameter AB. P is a point on AC such that the
circle center P radius PC touches S at T. Show that AP/AC = 4/5. Find AT/AC.

Answer

√(3/7)

Solution
Let O be the center of the circle. Let OA = R and PT = PC = r. Cosine rule applied to PAO
gives PO2 = PA2 + AO2 - 2PA·AO cos 60o, or (R+r)2 = (2R-r)2 + R2 - (2R-r)R. Hence r = 2R/5
and AP = 8R/5, so AP/AC = 4/5.

Applying cosine rule to APO and ATO, AP2 = OP2 + OA2 - 2OPOA cos x, AT2 = OT2 + OA2 -
2OTOA cos x, where x = ∠AOT. Hence AT2 = (2 - 7/5 - 5/7 + 74/35)R2 = 12R2/7. Hence
AT/AC = √(3/7).

Problem 2

Show that the number of ways of dividing {1, 2, ... , 2n} into n sets of 2 elements is 1.3.5 ...
(2n-1). There are 5 married couples at a party. How many ways may the 10 people be divided
into 5 pairs if no married couple may be paired together? For example, for 2 couples a, A, b,
B the answer is 2: ab, AB; aB, bA.

Answer

544

Solution

Any permutation gives a pairing. But 2nn! permutations give the same pairing because the
order of the two people in each of the n pairs is unimportant and the order of the n pairs is
unimportant. Hence (2n)!/2nn! = 1·3·5 ... (2n-1).

So there are 945 ways of pairing 10 people. If we pair a given married couple together, then
there are 1·3·5·7 = 105 ways. If we pair two given married couples together, then there are
1·3·5 = 15 ways. If we pair three given married couples together, then there are 3 ways. If we
pair 4 or 5 given couples together then there is just 1 way.

Hence by the principle of inclusion and exclusion, the number of ways or pairing so that no
married couple is paired is 945 - 5·105 + 10·15 - 10·3 + 5·1 - 1 = 544.

Problem 3
The real numbers a, b, c, x, y, z satisfy: x2 - y2 - z2 = 2ayz, -x2 + y2 - z2 = 2bzx, -x2 - y2 + z2 =
2cxy, and xyz ≠ 0. Show that x2(1 - b2) = y2(1 - a2) = xy(ab - c) and hence find a2 + b2 + c2 -
2abc (independently of x, y, z).

Answer

Solution

Adding the first two equations: -2z2 = 2z(ay+bx). But z ≠ 0, so bx + ay + z = 0. Similarly,


adding other pairs we get x + cy + bz = 0, cx + y + az = 0. Substituting z = -(bx+ay) in the
second two equations gives x(1-b2) = y(ab-c) and y(1-a2) = x(ab-c). Hence x2(1-b2) = y2(1-a2)
= xy(ab-c).

Multiplying x2(1-b2) = xy(ab-c) by y2(1-a2) = xy(ab-c) gives x2y2(1-a2)(1-b2) = x2y2(ab-c)2. But


x, y ≠ 0, so (1-a2)(1-b2) = (ab-c)2. Expanding gives a2+b2+c2-2abc = 1.

Thanks to Suat Namli

Problem 4

Find all positive integer solutions to 1/a + 2/b - 3/c = 1.

Answer

(a,b,c) = (2,1,2), (2,3,18), (1,2n,3n), (n,2,3n)

Solution

If a ≥ 3, then 2/b = 1 + 3/c - 1/a > 2/3, so b < 3. So either a < 3 or b < 3. So we consider 4
cases: a = 1, a = 2, b = 1, b = 2.

If a = 1, then 2/b = 3/c, so 2c = 3b. Hence b must be even. Put b = 2n, then c = 3n, which
gives a solution for any n.

If a = 2, then 2/b = 1/2 + 3/c > 1/2, so b ≤ 3. The cases b = 1, 2 we deal with below. If b = 3,
then c = 18, which gives a solution.
If b = 1, then 3/c = 1 + 1/a > 1, so c = 1 or 2. c = 1 does not give a solution. c = 2 gives a = 2,
which is a solution.

If b = 2, then 1/a = 3/c, so c = 3a. This gives a solution for any a.

Problem 5

L and M are skew lines in space. A, B are points on L, M respectively such that AB is
perpendicular to L and M. P, Q are variable points on L, M respectively such that PQ is of
constant length. P does not coincide with A and Q does not coincide with B. Show that the
center of the sphere through A, B, P, Q lies on a fixed circle whose center is the midpoint of
AB.

Solution

Let O be the midpoint of AB, and C be the center of the sphere. Then ∠COA = 90o, so C lies
in the plane p through O normal to AB. C also lies in the plane q normal to AP through its
midpoint. Let P', Q' be the feet of the perpendiculars from P, Q to p. Then the plane q meets
the plane p in the perpendicular bisector of OP'. So C must lie on this perpendicular bisector.
Similarly, C must lie on the perpendicular bisector of OQ', so it must be the circumcenter of
OP'Q'. But PP' = QQ' = OA, so PQ2 = P'Q'2 + AB2, so P'Q' has the fixed length √(PQ2-AB2).
The angle P'OQ' is also fixed, because if we take L' to be the line parallel to L through O and
M' to be the line parallel to M through O, then L' and M' lie in the plane p, P lies on L', and Q
lies on M'. Hence OC = P'Q'/(2 sin P'OQ'), which is fixed.

Problem 6

Show that if there are triangles with sides a, b, c, and A, B, C, then there is also a triangle with
sides √(a2 + A2), √(b2 + B2), √(c2 + C2).

Solution

We have to show that √(a2+A2) ≤ √(b2+B2) + √(c2+C2) (and two similar relations). Squaring,
that is equivalent to a2+A2 < b2+B2+c2+C2 + 2√((b2+B2)(c2+C2)). By Cauchy-Scwartz we have
bc + BC ≤ √((b2+B2)(c2+C2)), hence b2+B2+c2+C2 + 2√((b2+B2)(c2+C2)) ≥
b2+B2+c2+C2+2bc+2BC = (b+c)2 + (B+C)2 > a2 + A2.
25th BMO 1989 - Further International Selection Test

1. Find the smallest positive integer a such that ax2 - bx + c = 0 has two distinct roots in
the interval 0 < x < 1 for some integers b, c.

2. Find the number of different ways of arranging five As, five Bs and five Cs in a row so
that each letter is adjacent to an identical letter. Generalise to n letters each appearing five
times.

3. f(x) is a polynomial of degree n such that f(0) = 0, f(1) = 1/2, f(2) = 2/3, f(3) = 3/4, ... , f(n)
= n/(n+1). Find f(n+1). Solution

Put g(x) = (x+1)f(x)-x. Then g(0) = g(1) = ... = g(n) = 0, so g(x) = Ax(x-1)(x-2)...(x-n). g(-1)
= 1, so A = (-1)n+1/n+1!. Hence g(n+1) = (-1)n+1. So if n is odd, g(n+1) = 1 and f(n+1) = 1. If n
is even, then g(n+1) = -1 and f(n+1) = n/(n+2).

Thanks to Suat Namli

4. D is a point on the side AC of the triangle ABC such that the incircles of BAD and BCD
have equal radii. Express | BD | in terms of the lengths a = | BC |, b = | CA |, c = | AB |.

26th BMO 1990

Problem 1

Show that if a polynomial with integer coefficients takes the value 1990 at four different
integers, then it cannot take the value 1997 at any integer.

Solution

Let the polynomial be p(x) and the integers at which it is 1990 be a, b, c, d. Let q(x) = (x - a)
(x - b)(x - c)(x - d). Then p(x) - 1990 = q(x) r(x), where r(x) has integer coefficients. [We can
successively subtract off integral multiples of q(x) to zero the terms of p(x) - 1990 starting at
the highest until we get a polynomial of degree less than 4, but then that has 4 distinct roots
and so must be zero].
But now if p(k) = 1997, then 7 = q(k) r(k) = (k - a)(k - b)(k - c) r(k). At most two of the linear
factors can be ±1, so this is not possible.

Problem 2

The fractional part { x } of a real number is defined as x - [x]. Find a positive real x such that
{ x } + { 1/x } = 1 (*). Is there a rational x satisfying (*)?

Solution

There is no loss of generality in taking x > 1. So {1/x} = 1/x and {x} = x - n for some n. So
we have x - n + 1/x = 1 or x2 - (n+1)x + 1 = 0, so x = (n+1)/2 + 1/2 √(n2+2n-3). Evidently n =
0 and n = 1 do not work. But n = 2 gives x = (√5 + 3)/2, which works. Similarly, for example
n = 3 gives x = 2 + √3, which works. But √(n2+2n-3) cannot be integral (because n is too
small and n+1 is too large) and hence, by the usual argument, not rational either. So there are
no rational solutions.

Problem 3

Show that √(x2 + y2 - xy) + √(y2 + z2 - yz) ≥ √(z2 + x2 + zx) for any positive real numbers x, y,
z.

Solution

Note that there is a + on the rhs not a - .

A neat geometric approach is take OA = x, ∠AOB = 60o, OB = y, ∠BOC = 60o, OC = z.


Then, by the cosine rule, AB = √(x2 + y2 - xy), BC = √(y2 + z2 - yz) and AC = √(z2 + x2 + zx).

Problem 4

A rectangle is inscribed in a triangle if its vertices all lie on the boundary of the triangle.
Given a triangle T, let d be the shortest diagonal for any rectangle inscribed in T. Find the
maximum value of d2/area T for all triangles T.

Solution

Answer: (4√3)/7.
Suppose two vertices are on the side length a and that the corresponding altitude is length h.
Let the side parallel to the altitude have length x. Then the other side has length (h - x)a/h. So
the square of the diagonal is x2 + a2(1 - x/h)2 = (1 + a2/h2)(x - a2h/(a2+h2) )2 + a2h2/(a2+h2).
Hence d2 = a2h2/(a2+h2), or rather that is true provided we have picked the right side.

Suppose b is another side length with corresponding altitude k and. Let A = area T, then ah =
bk = 2A. So we require a2 + h2 ≥ b2 + k2 or a2 + 4A2/a2 ≥ b2 + 4A2/b2 or (a2 - b2)(1 - 4A2/(a2b2) )
≥ 0. But ab ≥ 2A (with equality iff the sides a and b are perpendicular, but the longest side is
never perpendicular to another side). So we get the shortest diagonal by taking the longest
side.

The required ratio d2/A is thus 2ah/(a2 + h2), where a is the longest side. We may assume A =
1/2 (since the ratio is unaffected by the scale of the triangles). So we have to maximise 2/(a2 +
h2) or equivalently to minimise a2 + 1/a2. The area of an equilateral triangle with side 1 is
(√3)/4 < 1/2, so a > 1, but a2 + 1/a2 is an increasing function of a for a > 1, so we want to
minimise a subject to the constraint that A is fixed. That obviously occurs for an equilateral
triangle. An equilateral triangle with side 1 has altitude (√3)/2, so d2/A has maximum value
√3/(1 + 3/4) = (4√3)/7.

Problem 5

ABC is a triangle with incenter I. X is the center of the excircle opposite A. Show that AI·AX
= AB·AC and AI·BX·CX = AX·BI·CI.

Solution

We show that triangles AIB and ACX are similar. Obviously ∠BAI = ∠XAC. ∠XIC = A/2 +
C/2, ∠ICX = 90o, so B/2 = ∠IXC = ∠AXC (same angle). Hence ∠ABI = ∠AXC, and the
triangles are similar. So AI/AB = AC/AX, which gives the first equation.

The same argument shows that AIC and ABX are similar. So BI/AI = XC/AC and CI/AC =
XB/AX. Multiplying gives the second equation.

27th BMO 1991


Problem 1

ABC is a triangle with ∠B = 90o and M the midpoint of AB. Show that sin ACM ≤ 1/3.

Solution

Take AM = MB = 1 and let BC = x. Then CM2 = x2 + 1, CA2 = x2 + 4. Applying the cosine


rule to triangle ACM, we get cos ACM = (2 + x2)/(x4 + 5x2 + 4)1/2. We claim that this is at
least 2√2)/3 with equality iff x = √2. Certainly angle ACM < 90o, so cos ACM is positive so
the claim is equivalent to 9(x4 + 4x2 + 4) ≥ 8(x4 + 5x2 + 4), or x4 - 4x2 + 4 ≥ 0, or (x2 - 2)2 ≥ 0,
which establishes the result. Thus the maximum value of sin ACM is (1 - 8/9)1/2 = 1/3.

Alternative solution

Let N be the midpoint of AC. Let G be the centroid (intersection of BN and CM). Let P be the
foot of the perpendicular from N to CM. Then sin ACM = NP/NC = NP/NB (angle B is 90o,
so N is the center of the circle ABC) = NP/3NG. But NP/NG ≤ 1 with equality iff BN and CM
are perpendicular.

Problem 2

Twelve dwarfs live in a forest. Some pairs of dwarfs are friends. Each has a black hat and a
white hat. Each dwarf consistently wears one of his hats. However, they agree that on the nth
day of the New Year, the nth dwarf modulo 12 will visit each of his friends. (For example, the
2nd dwarf visits on days 2, 14, 26 and so on.) If he finds that a majority of his friends are
wearing a different color of hat, then he will immediately change color. No other hat changes
are made. Show that after a while no one changes hat.

Solution

Consider the number of friendly pairs who wear a different color. It must increase if a dwarf
changes his hat. But it cannot increase indefinitely. [The details of the visiting rota are
irrelevant.]

Problem 3

A triangle has sides a, b, c with sum 2. Show that a2 + b2 + c2 + 2abc < 2.

Solution
Each side must be shorter than the sum of the other two. So a < 1, b < 1, c < 1. Hence 2(1 - a)
(1 - b)(1 - c) > 0, or 2 - 2(a + b + c) + 2(ab + bc + ca) - 2abc > 0, or -2 + 2(ab + bc + ca) - 2abc
> 0. Subtracting (a + b + c)2 = 4 gives the required result.

Problem 4

Let N be the smallest positive integer such that at least one of the numbers x, 2x, 3x, ... , Nx
has a digit 2 for every real number x. Find N. Failing that, find upper and lower bounds and
show that the upper bound does not exceed 20.

Solution

Answer: 12.

A real number may have infinitely many digits. The strategy therefore has to be to exhibit
some rational number which requires N to get a 2, and then to show that for any real number
some special digit, such as the first, is a 2 for some multiple <= N.

A little experimentation reveals that for most integers fairly small multiples suffice. We notice
that 15 requires 8, but most seem to need less.

If x has first digit 7, 8 or 9, then 3x has first digit 2. If x has first digit 5 or 6, then 4x has first
digit 2. If x has first digit 4, then 5x has first digit 2. If x has first digit 3, then 7x has first digit
2. Obviously, if x has first digit 2, then 1x has first digit 2. So we need only consider numbers
with first digit 1.

If x has second digit 2, we are home. If x has second digit 0 (including of course the case x
has no second digit), 1, 3 or 4, then 2x has first digit 2. If x has second digit 5, then 8x has
second digit 2.

It is convenient to notice that x and 10x behave the same way, so we can restrict x to be
between 1 and 10. The analysis above shows that we need only consider 1.6 ≤ x < 1.9. If 5/3 ≤
x < 2, then 20 ≤ 12x < 30.

We might also look at 5/3. Its multiples are 1.66... , 3.33... , 5, 6.66... , 8.33... , 10, 11.66... ,
13.33... , 15, 16.66... , 18.33... , 20. So it requires N = 12.

So we are left with 1.6 ≤ x < 5/3. The first digit is not 2 for N up to 12. But we notice that 2x
has a 2 for 1.6 ≤ x < 1.65, and for 1.66 ≤ x < 1.665 and for 1.666 ≤ x ≤ 1.6665 and so on. Also
5x has a 2 for 1.64 ≤ x < 1.66 and for 1.664 ≤ x < 1.666 and so on. Thus 2x or 5x has a 2 for
any x in the range 1.6 ≤ x < 5/3.

Note that there are other x which require N = 12, such as 1.6795 and 1.6835.
27th BMO 1991

Problem 1

ABC is a triangle with ∠B = 90o and M the midpoint of AB. Show that sin ACM ≤ 1/3.

Solution

Take AM = MB = 1 and let BC = x. Then CM2 = x2 + 1, CA2 = x2 + 4. Applying the cosine


rule to triangle ACM, we get cos ACM = (2 + x2)/(x4 + 5x2 + 4)1/2. We claim that this is at
least 2√2)/3 with equality iff x = √2. Certainly angle ACM < 90o, so cos ACM is positive so
the claim is equivalent to 9(x4 + 4x2 + 4) ≥ 8(x4 + 5x2 + 4), or x4 - 4x2 + 4 ≥ 0, or (x2 - 2)2 ≥ 0,
which establishes the result. Thus the maximum value of sin ACM is (1 - 8/9)1/2 = 1/3.

Alternative solution

Let N be the midpoint of AC. Let G be the centroid (intersection of BN and CM). Let P be the
foot of the perpendicular from N to CM. Then sin ACM = NP/NC = NP/NB (angle B is 90o,
so N is the center of the circle ABC) = NP/3NG. But NP/NG ≤ 1 with equality iff BN and CM
are perpendicular.

Problem 2

Twelve dwarfs live in a forest. Some pairs of dwarfs are friends. Each has a black hat and a
white hat. Each dwarf consistently wears one of his hats. However, they agree that on the nth
day of the New Year, the nth dwarf modulo 12 will visit each of his friends. (For example, the
2nd dwarf visits on days 2, 14, 26 and so on.) If he finds that a majority of his friends are
wearing a different color of hat, then he will immediately change color. No other hat changes
are made. Show that after a while no one changes hat.

Solution

Consider the number of friendly pairs who wear a different color. It must increase if a dwarf
changes his hat. But it cannot increase indefinitely. [The details of the visiting rota are
irrelevant.]

Problem 3
A triangle has sides a, b, c with sum 2. Show that a2 + b2 + c2 + 2abc < 2.

Solution

Each side must be shorter than the sum of the other two. So a < 1, b < 1, c < 1. Hence 2(1 - a)
(1 - b)(1 - c) > 0, or 2 - 2(a + b + c) + 2(ab + bc + ca) - 2abc > 0, or -2 + 2(ab + bc + ca) - 2abc
> 0. Subtracting (a + b + c)2 = 4 gives the required result.

Problem 4

Let N be the smallest positive integer such that at least one of the numbers x, 2x, 3x, ... , Nx
has a digit 2 for every real number x. Find N. Failing that, find upper and lower bounds and
show that the upper bound does not exceed 20.

Solution

Answer: 12.

A real number may have infinitely many digits. The strategy therefore has to be to exhibit
some rational number which requires N to get a 2, and then to show that for any real number
some special digit, such as the first, is a 2 for some multiple <= N.

A little experimentation reveals that for most integers fairly small multiples suffice. We notice
that 15 requires 8, but most seem to need less.

If x has first digit 7, 8 or 9, then 3x has first digit 2. If x has first digit 5 or 6, then 4x has first
digit 2. If x has first digit 4, then 5x has first digit 2. If x has first digit 3, then 7x has first digit
2. Obviously, if x has first digit 2, then 1x has first digit 2. So we need only consider numbers
with first digit 1.

If x has second digit 2, we are home. If x has second digit 0 (including of course the case x
has no second digit), 1, 3 or 4, then 2x has first digit 2. If x has second digit 5, then 8x has
second digit 2.

It is convenient to notice that x and 10x behave the same way, so we can restrict x to be
between 1 and 10. The analysis above shows that we need only consider 1.6 ≤ x < 1.9. If 5/3 ≤
x < 2, then 20 ≤ 12x < 30.

We might also look at 5/3. Its multiples are 1.66... , 3.33... , 5, 6.66... , 8.33... , 10, 11.66... ,
13.33... , 15, 16.66... , 18.33... , 20. So it requires N = 12.

So we are left with 1.6 ≤ x < 5/3. The first digit is not 2 for N up to 12. But we notice that 2x
has a 2 for 1.6 ≤ x < 1.65, and for 1.66 ≤ x < 1.665 and for 1.666 ≤ x ≤ 1.6665 and so on. Also
5x has a 2 for 1.64 ≤ x < 1.66 and for 1.664 ≤ x < 1.666 and so on. Thus 2x or 5x has a 2 for
any x in the range 1.6 ≤ x < 5/3.

Note that there are other x which require N = 12, such as 1.6795 and 1.6835.

28th BMO 1992

Problem 1

p is an odd prime. Show that there are unique positive integers m, n such that m2 = n(n + p).
Find m and n in terms of p.

Solution

We have (k2 + k)2 = k2(k2 + 2k + 1). So if p = 2k + 1, then m = k2 + k, n = k2 is always a


solution (whether or not p is prime).

4m2 = 4n2 + 4np, so 4m2 + p2 = (2n+p)2. Hence p2 = (2n+p+m)(2n+p-m). But p is prime, so the
only way of expressing p2 as a product of two unequal numbers is p2 x 1. Hence 2n+p-m = 1,
2n+p+m = p2. Solving for m and n, we get n = (p2 - 2p + 1)/4 = (p-1)2/4, m = (p2 - 1)/2.

Problem 2

Show that 12/(w + x + y + z) ≤ 1/(w + x) + 1/(w + y) + 1/(w + z) + 1/(x + y) + 1/(x + z) + 1/(y


+ z) ≤ 3(1/w + 1/x + 1/y + 1/z)/4 for any positive reals w, x, y, z.

Solution

The left-hand inequality is just the harmonic mean inequality applied to w+x, w+y etc. We
have 1/(w + x) + ... ≥ 62/(w + x + w + y + ... ) = 36/( 3(w + x + y + z) ).

The harmonic mean inequality applied to w and x gives 1/w + 1/x ≥ 4/(w + x). So applying it
to the other pairs also and adding we get 3(1/w + 1/x + 1/y + 1/z) ≥ 4(1/(w + x) + ... ), which
is the right-hand inequality.
Problem 3

The circumradius R of a triangle with sides a, b, c satisfies a2 + b2 = c2 - R2. Find the angles of
the triangle.

Solution

Answer: A = B = 30o and C = 120o.

We have a = 2R sin A etc (if O is the circumcenter, then ∠BOC = 2A). So we have that sin2A
+ sin2B = sin2C - 1/4. We try to simplify this. Put C = 180o - A - B, then sin C = sin(A + B) =
sin A cos B + cos A sin B. Hence we get sin2A + sin2B = sin2A cos2B + cos2A sin2B + 2 sin A
cos A sin B cos B - 1/4. Hence 2 sin2A sin2B = 2 sin A cos A sin B cos B - 1/4, or sin A sin B
(sin A sin B - cos A cos B) = -1/8 or sin A sin B cos C = -1/8.

It is not clear why this should determine A, B, C. We suspect that maybe sin A sin B cos C >=
-1/8 with equality only at the angles we want (that is a common ruse). sin A and sin B must be
positive, so cos C must be negative. For fixed A + B, we maximise sin A sin B by taking A =
B (for 2 sin A sin B = cos(A - B) - cos(A + B) ). So sin A sin B cos C >= sin2A cos(180o - 2A)
= - sin2A cos 2A = - sin2A (1 - 2 sin2A). To minimise this we must maximise k(1 - 2k), where
k = sin2A. Maximising k(1 - 2k) is equivalent to maximising 2k(1 - 2k) which is achieved by
taking 2k = 1 - 2k, so we want k = 1/4 and hence sin A = 1/2, so A = B = 30o and C = 120o.
The value then of sin A sin B cos C is indeed -1/8, so as predicted we have sin A sin B cos C
>= -1/8 with equality iff A = B = 30o and C = 120o. Since we are told that we do have
equality, the angles are fixed.

Problem 4

Each edge of a connected graph with n points is colored red, blue or green. Each point has
exactly three edges, one red, one blue and one green. Show that n must be even and that such
a colored graph is possible for any even n > 2. X is a subset of 1 < k < n points. In order to
isolate X from the other points (so that there is no edge between a point in X and a point not
in X) it is necessary and sufficient to delete R red edges, B blue edges and G green edges.
Show that R, B, G are all even or all odd.

Solution
The number of edges is 3n/2. This must be integral, so n must be even. Take a regular 2m-
gon. Make the sides alternately red and blue and draw the main diagonals (connecting each
point to the opposite point) green. Then each vertex has three edges, one of each color.

Suppose X has m vertices. There must be R red, B blue, and G green edges with just one end
a point of X. Suppose there are r red, b blue and g green edges with both ends at a point of X.
We have R + 2r = m, B + 2b = m, G + 2g = m. Hence R, B and G have the same parity.

Problem

The angles in the diagram below are measured in some unknown unit, so that a, b, ... , k, l are
all distinct positive integers. Find the smallest possible value of a + b + c and give the
corresponding values of a, b, ... , k, l.

Solution

Answer: 20.

We require the angles to satisfy: a + g + h = d + e + f = c + k + l = b + i + j = g + f + i = h + e


+ l = d + j + k = a + b + c. (One of these equations is dependent on the others.)

So we have to find some integer k which can be partitioned in many different ways as the sum
of three distinct integers. Suppose a + b + c + ... + l = n. The smallest possible value of n is 1
+ 2 + 3 + ... + 12 = 78 (since the numbers are all distinct). But note that each number occurs
twice in the equations above, so 8k = 2n, or n = 4k. So n must be a multiple of 4 and hence its
smallest value is 80. Note also that a + b + c = k, so minimising a + b + c is equivalent to
minimising n.

If n = 80, the only possibilities are {1, 2, 3, ... , 9, 10, 12, 13} or {1, 2, ... , 10, 11, 14}. At this
point it is not obvious how to proceed. A good deal of trial and error produces a = 1, b = 6, c =
13, d = 8, e = 3, f = 9, g = 7, h = 12, i = 4, j = 10, k = 2, l = 5. So there is a solution for k = 20,
so 20 is the required minimum.

Problem 2

p > 3 is prime. m = (4p - 1)/3. Show that 2m-1 = 1 mod m.

Solution

By Fermat, So 4p = 4 mod p, so 3m = 3 mod p. But p > 3 and prime, so m = 1 mod p. Also 4p -


1 is odd, so m is odd and hence m - 1 is even. So we may write m - 1 = 2kp. Hence 2m-1 = 4pk
= (3m + 1)k. Expanding by the binomial theorem, we see that (3m + 1)k = 1 mod m and hence
2m-1 = 1 mod m.

Problem 3

P is a point inside the triangle ABC. x = ∠BPC - ∠A, y = ∠CPA - ∠B, z = ∠APB - ∠C.
Show that PA sin A/sin x = PB sin B/sin y = PC sin C/sin z.

Solution

Extend AP, BP, CP to meet the circumcircle again at D, E, F respectively. Consider the
triangle PEC. We have ∠PEC = ∠BEC (same angle) = A (E on circumcircle). Also ∠BPC =
∠PEC + ∠ECP, so ∠ECP = x. Hence sin A/sin x = PC/PE. Similarly, sin B/sin y = PA/PF.
So we have to show that PA·PC/PE = PB·PA/PF or PC/PE = PB/PF. But CF and BE are
chords intersecting at P, so PC·PF = PB·PE.

The other equation follows similarly.

Problem 4

For 0 < m < 10, let S(m, n) is the set of all positive integers with n 1s, n 2s, n 3s, ... , n ms. For
a positive integer N let d(N) be the sum of the absolute differences between all pairs of
adjacent digits. For example, d(122313) = 1 + 0 + 1 + 2 + 2 = 6. Find the mean value of d(N)
for N in S(m, n).
Solution

Answer: n(m2 - 1)/3.

Consider the number of ways of placing two different digits h and k next to each other in
positions 1 and 2. The other mn - 2 digits can be permuted in (mn-2)! ways. But there are m-2
groups of n identical digits and 2 groups of n-1 identical digits, so the total number of ways is
(mn-2)! / (n!m-2 (n-1)!2). This is independent of the identity of h and k. There are 2(m-1) pairs
h, k with difference 1, 2(m-2) pairs with difference 2, and so on, up to 2 pairs with difference
m-1. Thus the total absolute difference between the first two digits (totalled over all members
of S(m, n) ) is 2( (m-1)1 + (m-2)2 + ... + 1(m-1) )(mn-2)! /( n!m-2 (n-1)!2) = f(m, n).

Exactly the same argument applies to the total absolute difference for positions 2 and 3, 3 and
4 and so on up to mn-1 and mn. Thus the total of d(N) over all members of S(m, n) is f(m, n)
(mn-1). There are (mn)! / n!m members of S(m, n), so the mean is f(m, n) (mn-1) n!m/ (mn)! .

We have (m-1)1 + (m-2)2 + ... + 1(m-1) = (m + 2m + 3m + ... + m.m) - (12 + 22 + ... + m2) =
m2(m+1)/2 - m(m+1)(2m+1)/6 = m(m+1)(3m - 2m - 1)/6 = m(m2 - 1)/6. Hence the mean is
1/3 m(m2 - 1) n!2/( (n-1)!2 mn) = n(m2 - 1)/3.

It is worth checking this for a few small values. Obviously the mean for S(1, n) = {11...1} is
zero, which checks. We have S(2, 1) = {12, 21}, mean 1, which checks. We have S(2, 2) =
{1122, 1212, 1221, 2112, 2121, 2211} mean 2, which checks.

As a further check, we found 2(m-1 + m-2 + ... + 1) (mn-2)! / (n!m-2 (n-1)!2) = m(m-1) (mn-2)!
/( n!m-2 (n-1)!2 ) numbers with different digits in positions 1 and 2. There are also
m.(mn-2)!/( n!m-1 (n-2)! ) numbers with the same first digit in positions 1 and 2. So in total we
have (mn-2)!/n!m (m(m-1) n2 + m n(n-1) ) = (mn-2)!/n!m mn ((m-1) n + n-1 ) = (mn)!/n!m as
required.

30th BMO 1994

Problem 1

Find the smallest integer n > 1 such that (12 + 22 + 32 + ... + n2)/n is a square.

Solution

We want (n+1)(2n+1)/6 = m2. But n+1 and 2n+1 are coprime. Also 2n+1 is odd, so we must
have either (1) 2n+1 = a2, n+1 = 6b2 or (2) 2n+1 = 3a2, n+1 = 2b2. But 2n+1 = 2(n+1) - 1, so in
case (1) we have a2 + 1 = 12b2. But squares are 0 or 1 mod 3, so a2 + 1 is 1 or 2 mod 3,
whereas 12b2 is 0 mod 3. So case (1) is not possible.

In case (2) we have 4b2 - 1 = 3a2, so either 2b + 1 = c2, 2b - 1 = 3d2 or 2b + 1 = 3d2, 2b - 1 =


c2, so c2 = 3d2 ± 2. We now try successively, d = 1, 2, ... . d = 1 gives c = 1, b = 1, n = 1. But
we are given n > 1. d = 2 fails, d = 3 gives a = 15, b = 13, n = 337.

Checking: (337 + 1)(2.337 + 1)/6 = 169.225 = (13.15)2.

Problem 2

How many incongruent triangles have integer sides and perimeter 1994?

Solution

Answer: 82834.

We assume a triangle is not allowed to be degenerate with zero area. Let tn be the number of
incongruent triangles with perimeter n and integer sides. Calculating the first few terms, we
find t1 = t2 = 0, t3 = 1 (111), t4 = 0, t5 = 1 (122), t6 = 1 (222), t7 = 2 (133, 223), t8 = 1 (233), t9 =
3 (144, 234, 333).

Suppose the triangle has sides a, b, c with a ≤ b ≤ c. The only restrictions are that a > 0 and a
+ b > c. Now consider a-1, b-1, c-1. This will be a triangle with perimeter 3 shorter unless a =
1 or a + b = c + 1. But if a = 1, then we must have b = c and hence a + b = c + 1. So a-1, b-1,
c-1 is either a triangle or a degenerate triangle. Conversely, if a-1, b-1, c-1 is a triangle or a
degenerate triangle, then a, b, c is a triangle. So if we put dn as the number of degenerate
triangles with perimeter n (including those with shortest side zero), then tn+3 = tn + dn.

Obviously dodd = 0. d2n = the number of pairs a ≤ b with a + b = n, which is just [n/2] + 1 (for
example 4 = 0 + 4 = 1 + 3 = 2 + 2). So we have t1994 = t1991 = t1988 + 498 = t1985 + 498 = t1982 +
496 + 498 = t1979 + 496 + 498 = t1976 + 495 + 496 + 498 = ... = 1 + 3 + 4 + 6 + 7 + ... + 496 +
498. There are 498 x 2/3 terms, which can be grouped into pairs with sum 499 (1 and 498, 3
and 496 etc), so the sum is 166 x 499.

Alternative solution

Assume a ≤ b ≤ c. We must have c < 1994/2, so the largest possible value of c is 996. Also c ≥
1994/3, so c ≥ 665. If c = 665, then the only solution is a = 664, b = 665. If c = 666, then the
solutions are a = 662, b = 666, or a = 663, b = 665, or a = b = 664. Evidently if c is odd, then
the number of pairs (a, b) is one greater than for c - 1 (if the smallest and largest possible a for
c-1 are h and k, then the largest and smallest for c are h-2 and k-1), but if c is even then the
number of pairs is two greater than for c-1 (the largest and smallest are h-2 and k). Hence the
total number of possibilities is 1 + 3 + 4 + 6 + 7 + ... + 498 (for c = 996 the possibilities are a
= 2 up to a = 499).
Problem 3

A, P, Q, R, S are distinct points on a circle such that ∠PAQ = ∠QAR = ∠RAS. Show that
AR(AP + AR) = AQ(AQ + AS).

Solution

Take X on the ray AQ so that QX = AS, and take Y on the ray AR so that RY = AP. Then we
have to show that AQ·AX = AR·AY, so it is sufficient to show that QRYX is cyclic.

We have QR = SR (because they subtend equal angles at A), QX = SA (by construction) and
∠RQX = 180o - ∠AQR = ∠RSA. So triangles QRX and SRA are congruent. So ∠YRX =
∠RXA + ∠RAX. But ∠RXA = ∠RXQ (same angle) = ∠RAS (congruent triangles). So
∠YRX = ∠RAX + ∠RAS = ∠RAX + ∠QAP (given as equal) = ∠PAR. But RX = AR
(QRX and SRA congruent) and RY = AP (by construction). So RYX and APR are congruent.
So ∠RYX = ∠APR = ∠AQR = 180o - ∠RQX. So QRYX is cyclic.

Problem 4

How many perfect squares are there mod 2n?

Solution

Answer: n odd (2n-1 + 5)/3, n even (2n-1 + 4)/3.

We find the first few:

2: 0, 1

4: 0, 1

8: 0, 1, 4

16: 0, 1, 4, 9

32: 0, 1, 4, 9, 16, 17, 25

64: 0, 1, 4, 9, 16, 17, 25, 33, 36, 41, 49, 57

a2 = b2 mod 2n iff (2a)2 = (2b)2 mod 2n+2, so the number of even squares mod 2n+2 equals the
number of squares mod 2n. This suggests looking separately at the odd and even squares. Let
un be the number of odd squares mod 2n and vn be the number of even squares mod 2n.
(2a + 1)2 = (2b + 1)2 mod 2n is equivalent to 4a2 + 4a = 4b2 + 4b mod 2n, which is equivalent to
a2 + a = b2 + b mod 2n-2 or (a - b)(a + b + 1) = 0 mod 2n-2. But a - b and a + b + 1 have sum 2a
+ 1 which is odd, so they have opposite parity, so either a - b = 0 mod 2n-2 or (a + b + 1) = 0
mod 2n-2. Thus just 3 other odd numbers have the same square as (2a + 1), namely (2a + 2n-1 +
1), -(2a + 1) and -(2a + 2n-1 + 1). So there are 2n-3 different odd squares mod 2n. In other words,
vn = 2n-3.

Now un = un-2 + vn-2 = un-4 + vn-4 + vn-2 etc. So the total number of squares u2m + v2m = v2m + v2m-2
+ v2m-4 + ... + v4 + u4 = 22m-3 + 22m-5 + ... + 2 + 2 = 2(4m-2 + 4m-3 + ... + 1) + 2 = 2(4m-1 - 1)/3 + 2
= (2n-1 + 4)/3.

Similarly, the total number of squares for n odd is (2n-1 + 5)/4.

31st BMO 1995

Problem

Find all positive integers a ≥ b ≥ c such that (1 + 1/a)(1 + 1/b)(1 + 1/c) = 2.

Solution

Answer: (a, b, c) = (5, 4, 3), (8, 3, 3), (7, 6, 2), (9, 5, 2), (15, 4, 2).

We have 1 + 1/a ≤ 1 + 1/b ≤ 1 + 1/c. So 2 ≤ (1 + 1/c)3. But (1 + 1/c)3 is a decreasing function


of c and c = 4 gives 125/64 < 2. So the only possible values of c are 1, 2, 3. If c = 1, then (1 +
1/a)(1 + 1/b) = 1, which is obviously impossible. So c = 2 or 3.

We may write the expression as (c + 1)(a + b + 1) = (c - 1)ab. Suppose c = 2. Then (b - 3)a =


3b + 3. If b ≥ 7, then (b - 3)a ≥ 4a ≥ 4b ≥ 3b + 7 > 3b + 3. Contradiction. (b - 3)a = 3b + 3, so
b - 3 must be positive. Thus we need only consider b = 4, 5 and 6. These give the three
solutions with c = 2 shown above.

Suppose c = 3. Then 2ab = 4(a + b + 1), so ab = 2a + 2b + 2. So (b - 2)a = 2b + 2. If b ≥ 5,


then (b - 2)a ≥ 3a ≥ 3b ≥ 2b + 5 > 2b + 2. Contradiction. But b - 2 must be positive, so we
need only consider b = 3 and 4. That gives the two solutions shown above for c = 3.
Problem 2

ABC is a triangle. D, E, F are the midpoints of BC, CA, AB. Show that ∠DAC = ∠ABE iff
∠AFC = ∠ADB.

Solution

The medians meet at the centroid G. DF is parallel to AC, so ∠FDG = ∠DAC (alternate
angles) = ∠ABE (given) = ∠FBG (same angle). So FGDB is cyclic. So ∠ADB = ∠GDB
(same angle) = 180o - ∠GFB = ∠AFC.

Problem 3

x, y, z are real numbers such that x < y < z, x + y + z = 6 and xy + yz + zx = 9. Show that 0 <
x < 1 < y < 3 < z < 4.

Solution

(x + y) = 6 - z, so 9 = xy + yz + zx = xy + z(6 - z). Hence (z - 3)2 = xy. Similarly, (x - 3)2 = yz


and (y - 3)2 = zx. If any of x, y, z are zero, then two of (z - 3)2, (y - 3)2 and (x - 3)3 are zero, so
two of x, y, z are equal, whereas we are told they are all unequal. Hence none of x, y, z are
zero.

We also have that xy, yz and zx are all non-negative. So we cannot have x < 0 and y and z > 0
(for then xy < 0). Equally, we cannot have y < 0 and z > 0 (for then yz < 0). Finally, we
cannot have x, y, z all negative, for then x + y + z would be negative. So x, y, z are all
positive.

We have x + y = 6 - z. Also xy < (x + y)2/4 (strict inequality since x and y are unequal). So (6
- z)2/4 + (6 - z)z > 9 or 4z - z2 > 0. Hence z < 4. If z ≤ 2, then since x < y < z, we have x + y +
z < 6. Contradiction. hence z > 2. So |z - 3| < 1. But xy = (z - 3)2, so xy < 1. So x < 1.

But (x - 3)2 = yz, so yz > 4. But z < 4, so y > 1. If y ≥ 3, then x + y + z > y + z > 2y > 6.
Contradiction. So y < 3.

Finally, (x - 3)(y - 3)(z - 3) = xyz - 3(xy + yz + zx) + 9(x + y + z) - 27 = xyz > 0. But (x - 3)(y
- 3) > 0, so z > 3.

Problem 4

(1) How many ways can 2n people be grouped into n teams of 2?


(2) Show that (mn)! (mn)! is divisible by m!n+1 n!m+1 for all positive integers m, n.
Solution

(1) Let the number of ways be t(n). Then t(1) = 1 and t(n) = (2n-1) t(n-1). So t(n) = 1.3.5 ...
(2n-1). We can also write this as (2n)! /(2.4.6 ... 2n) = (2n)! / (2nn!).

(2) The first part is evidently intended as a hint. Consider the number of ways of dividing mn
people into m teams of n people. It is the number of ways of arranging mn people into a line
divided by the number of ways of ordering the m teams times the number of ways or ordering
each team = (mn)! / ( n! m!n ). So this must be integral. Similarly, (mn)! / ( m! n!m ) is integral
and hence their product.

32nd BMO 1996

Problem 1

Find all non-negative integer solutions to 2m + 3n = k2.

Solution

Answer: 20 + 31 = 22, 23 + 30 = 32 , 24 + 32 = 52.

If m = 0, then 3n = k2 - 1 = (k + 1)(k - 1). But if 3 divides both k + 1 and k - 1, then it divides


their difference 2, which is impossible, so k - 1 = 1 and we have the solution m = 0, n = 1, k =
2.

If m = 1, then k2 - 2 = 3n. But squares are 0 or 1 mod 3, so k2 - 2 is 1 or 2 mod 3, so n must be


0. But that does not give a solution. So we may assume m > 1. Hence k2 = 3n = (4 - 1)n = (-1)n
mod 4. But squares are 0 or 1 mod 4, so n must be even. Put n = 2s. Then 2m = k2 - 32s = (k +
3s)(k - 3s). So both k + 3s and k - 3s are powers of 2. Suppose k - 3s = 2a and k + 3s = 2b, so that
m = a + b. Then 2.3s = 2b - 2a. So a = 1 and 3s = 2b-1 - 1. But 2b-1 - 1 = (-1)b-1 - 1 mod 3, so b - 1
must be even or s = 0.

If s = 0, then b - 1 = 1, so m = 3 and we have the solution 23 + 30 = 32.

So suppose b -1 is even. Put b - 1 = 2r, then 3s = (2r - 1)(2r + 1). But 3 cannot divide both 2r - 1
and 2r + 1 or it divides their difference, which is 2. So we must have 2r - 1 = 1 and hence r = s
= 1, b = 2r+1 = 3, n = 2s = 2, m = a + b = 4 and we have the solution 24 + 32 = 52.
Problem 2

The triangle ABC has sides a, b, c, and the triangle UVW has sides u, v, w such that a2 = u(v
+ w - u), b2 = v(w + u - v), c2 = w(u + v - w). Show that ABC must be acute angled and
express the angles U, V, W in terms of the angles A, B, C.

Solution

Thanks to Arne Smeets

a2 + b2 - c2 = u(v + w - u) + v(w + u - v) - w(u + v - w) = w2 - (u - v)2 = 2uv(1 - cos W) > 0.


Similarly, b2 + c2 - a2 > 0, and c2 + a2 - b2 > 0. So ABC is acute-angled.

We have ab = √(uv(w2 - u2 - v2 + 2uv) ) = uv √( 2(1 - cos W) ), so cos C = (a2 + b2 - c2)/2ab =


√ ((1 - cos W)/2 ) = sin W/2 = cos(90o - W/2). Thus W = 180o - 2C. Similarly for the other
angles.

Problem 3

Thanks to Arne Smeets

The circles C and C' lie inside the circle S. C and C' touch each other externally at K and
touch S at A and A' respectively. The common tangent to C and C' at K meets S at P. The line
PA meets C again at B, and the line PA' meets C' again at B'. Show that BB' is a common
tangent to C and C'.

Solution
Let the centers of C, C' be O, O'. We have PA·PB = PK2 = PA'·PB', so triangles PAA' and
PB'B are similar. Hence ∠PA'A = ∠PBB'. Let W be the center of S. A homothecy center A
takes O to W, C to S and B to P, so ∠BOA = ∠PWA. Hence ∠PBB' = ∠PA'A = ½ ∠PWA =
½ ∠BOA = ∠BOM, where M is the midpoint of AB.

∠OMB = 90o, so ∠OBB' = 180o - ∠PBB' - ∠MBO = 90o. Hence BB' is tangent to C.
Similarly, it is tangent to C'.

Problem 4

Find all positive real solutions to w + x + y + z = 12, wxyz = wx + wy + wz + xy + xz + yz +


27.

Answer

w = x = y = z = 3 is the only solution.

Solution

Put p = √(wxyz). By AM/GM applied to wx, wy, wz, xy, xz, yz we have p ≤ (wx + wy + wz +
xy + xz + yz)/6 = p2/6 - 27/6, so (p - 3)2 ≥ 36. Hence |p - 3| ≥ 6. But p is positive, so p ≥ 9.
Hence wxyz ≥ 81. But applying AM/GM to w, x, y, z we have √p ≤ 3, with equality iff w = x
= y = z. So wxyz ≤ 81 with equality iff w = x = y = z.

33rd BMO 1997

Problem 1

M and N are 9-digit numbers. If any digit of M is replaced by the corresponding digit of N (eg
the 10s digit of M replaced by the 10s digit of N), then the resulting integer is a multiple of 7.
Show that if any digit of N is replaced by the corresponding digit of M, then the resulting
integer must be a multiple of 7. Find d > 9, such that the result remains true when M and N
are d-digit numbers.
Solution

Let M = m8108 + m7107 + ... + m0 and N = n8108 + ... + n0. We have:


n8108 + m7107 + m6106 + ... + m110 + m0 = 0 mod 7
m8108 + n7107 + m6106 + ... + m110 + m0 = 0 mod 7
m8108 + m7107 + n6106 + ... + m110 + m0 = 0 mod 7
...
m8108 + m7107 + m6106 + ... + n110 + m0 = 0 mod 7
m8108 + m7107 + m6106 + ... + m110 + n0 = 0 mod 7

Adding all but the first equation we get:


8m8108 + (n7 + 7m7) 107 + (n6 + 7m6) 106 + ... + (n0 + 7m0) = 0 mod 7. Hence
m8108 + n7107 + n6106 + ... + n0 = 0 mod 7, which shows that substituting the first digit from M
into N gives a number divisible by 7.

Similarly, adding all but the second equation, we find that substituting the second digit from
M into N gives a number divisible by 7, and so on.

Obviously the same argument works for any d = 2 mod 7.

Problem 2

ABC is an acute-angled triangle. The median BM and the altitude CF have equal length, and
∠MBC = ∠FCA. Show that ABC must be equilateral.

Solution

Solution by Paul Smith

Put ∠MBC = ∠FCA = x. Since M is the midpoint of AC and ∠AFC = 90o, M must be the
center of the circle through A, F, C, so MF = MC = MA. Put MF = k. Then CF = 2k cos x.

But ∠MBC = ∠FCA = ∠FCM = ∠MFC, so MFBC is cyclic. Hence ∠BMC = ∠CFB = 90o.
So BM/k = cot MBC = cot x. But CF = BM, so sin x = 1/2. Hence x = 30o. Hence ∠C = 60o.
Also ∠A = 90o - angle FCA = 60o. So the triangle is equilateral.

I originally had this appalling slog:

Let the side lengths be BC = a, CA = b, AB = C and let CF = BM = m. Let ∠MBC = ∠FCA


= x. By the cosine formula we have: c2 = m2 + b2/4 - mb cos BMA, a2 = m2 + b2/4 + mb cos
BMA. Adding, m2 = a2/2 + c2/2 - b2/4 (1).

From triangle FCA, m = b cos x. By the cosine formula to MBC, b2/4 = m2 + a2 - 2am cos x.
Hence b2/4 = m2 + a2 - 2am2b = (b - 2a)m2/b + a2. Hence b = 2a or m2 = b(b + 2a)/4.
Suppose b = 2a. If m <= a, then B lies inside or on the circle center M radius MA and hence
∠ABC ≥ 90o. But we are told the triangle is acute-angled, so ∠ABC < 90o. Hence m > a. But
CF ≤ CB = a. Contradiction. So we cannot have b = 2a.

Hence m2= b(b + 2a)/4, so from (1), 2a2 + 2c2 - b2 = b2 + 2ab or b2 = a2 + c2 - ab.

Let k be the area of ABC. By Heron's formula we have that 16k2 = (a + b + c)(a + b - c)(a - b
+ c)(-a + b + c) = - a4 - b4 - c4 + 2a2b2 + 2b2c2 + 2c2a2. But 16k2 is also 4m2c2 = c2b(b + 2a). So
we have -a4 - b4 + 2a2b2 + c2(2a2 - 2ab + b2) - c4 = 0. Substituting for c2 ( = b2 - a2 + ab) we get:
0 = -4a4 - b4 + 2a2b2 - 3b3a + 6a3b = (a - b)(2a + b)(2a2 - 2ab - b2).

Clearly 2a + b is non-zero. If 2a2 - 2ab - b2 = 0, then 3a2 = (a + b)2, so b = (√3 - 1)a. Then
substituting in the previous relation for c, we get c = b/√2. So we have for some k, a = (1 +
√3)k, b = 2k, c = (√2)k. But then a2 > b2 + c2, so the triangle is not acute-angled. So we must
have a = b. Then using the previous relation for c, we get c = a also.

Problem 3

Find the number of polynomials of degree 5 with distinct coefficients from the set {1, 2, ... ,
9} which are divisible by x2 - x + 1.

Solution

Answer: 624.

This seems to be a laborious exercise in counting. Suppose that the polynomial is (x2 - x + 1)
(ax3 + bx2 + cx + d). Multiplying out, we find that the coefficients are: a, b-a, c-(b-a), b-(c-d),
c-d, d. Put e = b-a, f = c-d. Then the coefficients are: a, e, d+f-e, a+e-f, f, d. Put k = e-f. Then
the coefficients are: a, f+k, d-k, a+k, f, d. Suppose k > 0. Put g = d-k. then the coefficients are
(rearranging the order) a, f, g, a+k, f+k, g+k. Similarly, if k < 0, put h = -k, and the
coefficients are a, e, d+h, a-h, e+h, d. Putting g = a-h and rearranging the order we get: d, e, g,
d+h, e+h, g+h. The two types (k positive and negative) obviously do not overlap and there are
evidently equal numbers of each. So we have to find the number of sets a, f, g, a+k, f+k, g+k.
By symmetry, the number of sets must be 6 times the number with a < f < g. However, I
cannot see an elegant way to count the number with a < f < g. Simply listing them gives 52
possibilities.

if k = 1, a = 1, f = 3, then g can be 5, 6, 7, or 8.

if k = 1, a = 1, f = 4, then g can be 6, 7 or 8

if k = 1, a = 1, f = 5, then g can be 7 or 8

if k = 1, a = 1, f = 6, then g must be 8

if k = 1, a = 2, f = 4, then g can be 6, 7 or 8
if k = 1, a = 2, f = 5, then g can be 7 or 8

if k = 1, a = 2, f = 6, then g must be 8

if k = 1, a = 3, f = 5, then g can be 7 or 8

if k = 1, a = 3, f = 6, then g must be 8

if k = 2, a = 1, f = 2, then g can be 5, 6 or 7

if k = 2, a = 1, f = 4, then g can be 5 or 7

if k = 2, a = 1, f = 5, then g must be 6

if k = 2, a = 1, f = 6, then g must be 7

if k = 2, a = 2, f = 3, then g can be 6 or 7

if k = 2, a = 2, f = 5, then g must be 6

if k = 2, a = 2, f = 6, then g must be 7

if k = 2, a = 3, f = 4, then g must be 7

if k = 2, a = 3, f = 6, then g must be 7

if k = 3, a = 1, f = 2, then g can be 3 or 6

if k = 3, a = 1, f = 3, then g must be 5

if k = 3, a = 1, f = 5, then g must be 6

if k = 3, a = 2, f = 3, then g must be 4

if k = 3, a = 2, f = 4, then g must be 6

if k = 3, a = 3, f = 4, then g must be 5

if k = 3, a = 4, f = 5, then g must be 6

if k = 4, a = 1, f = 2, then g can be 3 or 4

if k = 4, a = 1, f = 3, then g must be 4

if k = 4, a = 2, f = 3, then g can be 4 or 5

if k = 4, a = 2, f = 4, then g must be 5

if k = 5, a = 1, f = 2, then g must be 3 or 4

if k = 5, a = 1, f = 3, then g must be 4

if k = 5, a = 2, f = 3, then g must be 4

if k = 6, a = 1, f = 2, then g must be 3

Comment: can anyone come up with a better solution!


Problem 4

Let S be the set {1/1, 1/2, 1/3, 1/4, ... }. The subset {1/20, 1/8, 1/5} is an arithmetic
progression of length 3 and is maximal, because it cannot be extended (within S) to a longer
arithmetic progression. Find a maximal arithmetic progression in S of length 1996. Is there a
maximal arithmetic progression in S of length 1997?

Solution

Let k = 1/1993! . Then h divides k for h = 1, 2, ... , 1996, but not for 1997, which is prime.
Hence 1/k, 2/k, 3/k, ... , 1996/k is a maximal arithmetic progression in S of length 1996
(extending it would require 0 or 1997/1993! to be in S, which they are not).

6.1998 - 1 = 11987 is prime. So put k = 11986! and consider the progression 5/k, 11/k,
17/k, ... , 11981/k. It has 1997 terms in S. It cannot be extended, because -1/k and 11987/k are
not in S.

[This conceals some work, because we have to try 2·1998 - 1 (factor 5), 3·1998 - 1 (factor
13), 3·1998 - 2 (factor 2), 4·1998 - 1 (factor 61), 4·1998 - 2 (factor 2), 4·1998 - 3 (factor 3),
5·1998 - 1 (factor 7), 5·1998 - 2 (factor 2), 5·1998 - 3 (factor 3), 5·1998 - 4 (factor 2), before
hitting on 6·1998 - 1 (and checking that requires checking divisibility by about 25 primes).]

BMO 1998 Round 2

Problem 1

A station issues 3800 tickets covering 200 destinations. Show that there are at least 6
destinations for which the number of tickets sold is the same. Show that this is not necessarily
true for 7.

Solution

There is some ambiguity as to whether the station can issue 0 tickets for a particular
destination. Suppose 5 destinations had 0 tickets, then would it be true that the station issued
"3800 tickets covering 200 destinations"? It would be more natural to say 3800 tickets
covering 195 destinations. However, the point is not important, because the result is true
either way.

If we allow zero, then the smallest possible number of tickets that could be sold if no more
than 5 destinations had the same number of tickets would be 5(1 + 2 + ... + 39) = 5·39·40/2 =
3900. The number of tickets would obviously be higher if we did not allow zero. In any case,
only 3800 tickets were sold, so at least 6 destinations have the same number of tickets.

However, 6(1 + 2 + ... 33) + 34 + 400 = 6·33·34/2 + 434 = 3800 and 6·33 + 2 = 200, so there
could be 6 destinations with just 1 ticket, 6 with just 2 tickets and so on, up to 6 with just 33
tickets, then 1 destination with 34 tickets and 1 with 400 tickets. In this case there is no group
of 7 or more destinations with the same number of tickets.

Problem 2

The triangle ABC has ∠A > ∠C. P lies inside the triangle so that ∠PAC = ∠C. Q is taken
outside the triangle so that BQ parallel to AC and PQ is parallel to AB. R is taken on AC (on
the same side of the line AP as C) so that ∠PRQ = ∠C. Show that the circles ABC and PQR
touch.

Solution

Let the lines BQ and AP meet at X. We show that the circles touch at X.

By construction ∠PAC = ∠C. BQ is parallel to AC, so ∠BXA = ∠C. Also by construction


∠PRQ = ∠C. So ∠PRQ = ∠PXQ and hence X lies on the circle PQR. But since ∠BXA
(another name for ∠PXQ) = ∠C, X also lies on the circle ABC.

Finally, since PQ is parallel to AB, ∠BAX = ∠QPX, so the angles made by the tangents at X
to the circles ABC and PQR with the chord BQX are the same. Hence the tangents coincide
and the circles touch at X.

Problem 3

a, b, c are positive integers satisfying 1/a - 1/b = 1/c and d is their greatest common divisor.
Prove that abcd and d(b - a) are squares.

Solution

Let A = a/d, B = b/d, C = c/d, so that A, B, C have greatest common divisor 1. We have 1/A -
1/B = 1/C, so AB = C(B - A). We show that (B - A) must be a square.

If not, then there is a prime p such that the highest power of p dividing (B - A) is p2r+1 for
some r. Now if pr+1 divides A, then it also divides B = (B - A) + A. Hence p2r+2 divides AB.
But p cannot divide C (since A, B, C have no common factor), so p2r+2 divides (B - A).
Contradiction. So at most pr divides A, and similarly B. Hence at most p2r divides AB and
hence (B - A). Contradiction. That establishes that (B - A) is a square. But ABC(B - A) =
ABAB, so ABC(B - A) is a square, and hence also ABC(B - A)/(B - A) = ABC.

Hence (b - a)d = d2(B - A) and abcd = d4ABC are squares.

Problem 4

Show that:

xy + yz + zx = 12
xyz - x - y - z = 2

have a unique solution in the positive reals. Show that there is a solution with x, y, z distinct
reals.

Solution

Obviously 2, 2, 2 is a solution (in the positive reals). If we regard z as known, then we have x
+ y = (11z - 2)/(z2 + 1), xy = (z2 + 2z + 12)/(z2 + 1). So necessary and sufficient conditions for
x, y, z to be positive reals are: (11z - 2)2 > 4(z2 + 2z + 12)(z2 + 1) (*) (which ensures that x
and y are real) and z > 2/11 (which then ensures that they and z are positive).

(*) becomes 4z4 + 8z3 - 69z2 + 52z + 44 ≤ 0, or (z - 2)2(2z + 11)(2z + 1) ≤ 0. This is satisfied
for -11/2 ≤ z ≤ -1/2 and z = 2. So the only positive solution is z = 2 (and hence x = 2, y = 2
also, since the problem is symmetrical between x, y and z).

For other real solutions we should look at values in the range -11/2 to -1/2. z = -1 does not
help (we get the non-distinct -1, -1, -11/2). z = -2 gives x = - (12 + 2√21)/5, y = - (12 -
2√21)/5, which are real and distinct.

A better solution to the first part was provided by Arne Smeets

So assume x, y, z are postive and put w = (xyz)1/3. Using AM/GM on xy + yz + zx = 12, we


get 12 ≥ 3 w2, so w ≤ 2. Using AM/GM on the other equation we get w3 ≥ 2 + 3w, or (w - 2)
(w + 1)2 >= 0, so w ≥ 2. Hence w = 2. But we only get equality in AM/GM if the terms are
equal, so x = y = z = 2.

35th BMO 1999


Problem 1

Let Xn = {1, 2, 3, ... , n}. For which n can we partition Xn into two parts with the same sum?
For which n can we partition Xn into three parts with the same sum?

Solution

The sum of all the elements of Xn is n(n+1)/2. This is odd for n = 1, 2 mod 4 and even for n =
0, 3 mod 4, so a necessary condition for two parts is that n = 0 or 3 mod 4. It is also sufficient.
For n = 0 mod 4, there are an even number of pairs (1, n), (2, n-1), (3, n-2), ... each with the
same sum, so we can take half the pairs in one part and half in the other. For n = 3 mod 4,
there are an even number of pairs (4, n), (5, n-1), (6, n-2), ... each with the same sum, so we
take half in one part and half in the other. Then we put 1 and 2 in one part and 3 in the other.

A necessary condition is that 3 divides n(n+1), so n is not 1 mod 3. That is not sufficient,
because we obviously cannot divide X2 or X3 into three equal parts. However, it is sufficient
for larger n.

Let N = n(n+1)/6. For n not 1 mod 3, this is an integer. Take a subset A of Xn by using the
greedy algorithm: we repeatedly pick the largest remaining element until we get the required
sum N. Thus A consists of a sequence of consecutive integers ending with n and possibly one
additional element. Now take another subset B of the remaining elements, using the same
algorithm. The remaining elements are 1, 2, ... , m for some m with at most one element
missing. The algorithm cannot fail for A and can only fail for B if either: (1) the missing
element is 1 and after picking m, m-1, ... , m-k we want 1 to complete the set, or (2) the
missing element is 2, and after picking m, m-1, ... , m-k we want 2 to complete the set (we can
continue with 1 but then get stuck). In case (1), we can use replace m-k and pick m-k-1 and 2.
In case (2) we can replace m-k (and 1 if we got that far) and pick m-k-1 and 3.

Thus we can always get two sets with sum N. But then the remaining elements must also have
sum n.

Comment. This is a weaker version of the result that if 1 + 2 + 3 + ... + n = mk, with n ≥ m,
then we can divide the set into k parts with equal sum. See ASU 88/20

Problem 2

A circle is inscribed in a hexagon ABCDEF. It touches AB, CD and EF at their midpoints (L,
M, N respectively) and touches BC, DE, FA at the points P, Q, R. Prove that LQ, MR, NP are
concurrent.
Solution

The key is that PN is perpendicular to LM. So the three lines are the altitudes of the triangle
LMN and hence concurrent.

Let the center of the circle be O. Let ∠AOL = x, so ∠LOB = x (L midpoint of AB), ∠BOP =
x also (BP = BL) and ∠ROA = x. Similarly, let ∠POC = y. So ∠COM = y (CM = CP),
∠DOM = y (M midpoint of CD), ∠DOQ = y (DM = DQ). Similarly, let ∠QOE = z, so
∠EON = z (EN = EQ), ∠NOF = z (N midpoint of EF), ∠FOR = z (FN = FR). Thus 4(x + y +
z) = 360o and hence x + y + z = 90o.

∠LOM = 2x + 2y, so if OX is the perpendicular bisector of LM, then ∠LOX = x + y.


Similarly, ∠PON = 4y + 2z, so if OY is the perpendicular bisector of PN, then angle POY =
2y + z. ∠LOP = 2x, so ∠LOY = 2x + 2y + z. Hence ∠XOY = 2x + 2y + z - (x + y) = x + y +
z = 90o. In other words the perpendicular bisectors of LM and PN are perpendicular. Hence
LM and PN are perpendicular, which is all we need. [Obviously, a similar argument
establishes that that LQ, MR are also altitudes of LMN.]

Problem 3

Show that xy + yz + zx ≤ 2/7 + 9xyz/7 for non-negative reals x, y, z with sum 1.

Solution

If x ≥ 7/9, then 1 ≤ 9x/7, so xy ≤ 9xyz/7. Also (y + z) ≤ 2/9, so xy + xz < 2/9 < 2/7. Thus in
this case xy + yz + zx < 2/7 + 9xyz/7. So assume x < 7/9, and hence 1 - 9x/7 > 0.

We have yz ≤ (y+z)2/4 with equality iff y = z and hence yz ≤ (1 - x)2/4. So it is sufficient to


show that (1 - 9x/7)(1 - x)2/4 + x(1 - x) ≤ 2/7, or equivalently (7 - 9x)(1 - x)2 + 28x(1 - x) <=
8. Expanding, 9x3 + 3x2 - 5x + 1 >= 0 or (x + 1)(3x - 1)2 ≥ 0, which is obviously true (for non-
negative x), with equality iff x = 1/3. Thus the inequality holds, with equality iff x = y = z =
1/3.

Problem 4

Find the smallest possible sum of digits for a number of the form 3n2 + n + 1 (where n is a
positive integer). Does there exist a number of this form with sum of digits 1999?

Solution

Answer: n = 8 gives 201 with sum of digits 3. Yes, eg take n = 10222 - 1.


The only one digit number has sum of digits 5. 3n2 + n + 1 = n(3n+1) + 1 and n and 3n+1
have opposite parity, so 3n2 + n + 1 must be odd. In particular, the last digit cannot be 0, so
for n > 1 the number has at least two non-zero digits (the first and the last) and hence the sum
of digits is at least 2. It can only be 2 if the number is 10a + 1 and hence n(3n+1) = 10a. But n
and 3n+1 are relatively prime, so we must have n = 2a, 3n+1 = 5a. But (5/2)a > 6 for a > 1, so
there are no solutions with a > 1. If a = 1, then n = 2, so 3n+1 = 7 not 5, so a = 1 is not a
solution either. Thus the sum of digits cannot be 2.

n = 8 gives 201 with sum of digits 3, so that is the best possible.

We claim that if n = 10m - 1, then 3n2 + n + 1 = 29...950...03, where there are m-1 9s and m-1
0s. Thus if we take m = 222, then we get a number with 221 9s, one 2, one 3 and one 5, so the
sum of digits is 1999.

To prove the assertion, note that 3n2 + n + 1 = 3·102m - 6·10m + 3 + 10m = (3·10m-1 - 1) 10m+1 +
5·10m + 3. Now 3·10m-1 - 1 is an m-digit number 29...9. Multiplying by 10m+1 means it is
followed by m+1 zero digits. But 5·10m + 3 is an m+1 digit number 50...03, so the sum is as
claimed.

36th BMO 2000

Problem 1

Two circles meet at A and B and touch a common tangent at C and D. Show that triangles
ABC and ABD have the same area.

Solution

Let the common tangent meet the line AB at X. We have XC2 = XA·XB = XD2. So C and D
are equidistant from the line AB.

Problem 2

Find the smallest value of x2 + 4xy + 4y2 + 2z2 for positive reals x, y, z with product 32.

Solution

Answer: 96.
We have (x + 2y)2 ≥ 8xy with equality iff x = 2y. Hence (x + 2y)2 + 2z2 ≥ 256/z + 2z2 with
equality iff x = 2y. We have z2 + 128/z - 48 = (z - 4)2(z + 8)/z ≥ 0 with equality iff z = 4 (for
positive z). Hence 256/z + 2z2 ≥ 96 with equality iff z = 4. So the expression given has
minimum value 96, achieved only at x = 4, y = 2, z = 4.

Problem 3

Find positive integers m, n such that (m1/3 + n1/3 - 1)2 = 49 + 20 (61/3).

Solution

A little experimentation gives (2·61/3 + 2·62/3 - 1)2 = 49 + 20·61/3. So we can take m = 48, n =
288.

Problem 4

Find a set of 10 distinct positive integers such that no 6 members of the set have a sum
divisible by 6. Is it possible to find such a set with 11 members?

Solution

Thanks to Arne Smeets

Take five integers = 0 mod 6 and five = 1 mod 6. Then any six must include at least one and
at most five = 1 mod 6 and hence their sum cannot be 0 mod 6.

Amongst any 5 numbers there must be 3 with sum = 0 mod 3. (If three numbers have the
same residue, then their sum = 0 mod 3. If not, then there must be at least one for each
residue, and 0 + 1 + 2 = 0 mod 3.) So given 11 numbers, we can pick a1, a2, a3 with a1 + a2 + a3
= 0 mod 3. From the remaining 8, we can pick another three: b1, b2, b3. From the remaining 5
we can pick another three: c1, c2, c3. Two of the sums a1 + a2 + a3, b1 + b2 + b3, c1 + c2 + c3
must have the same parity, so their sum is even. Thus we have 6 numbers with sum = 0 mod
6.

37th BMO 2001


Problem 1

A has a marbles and B has b < a marbles. Starting with A each gives the other enough marbles
to double the number he has. After 2n such transfers A has b marbles. Find a/b in terms of n.

Solution

After 2 transfers A has 2(a - b) = 4a - 2N marbles, where N = a + b. Suppose A has an marbles


after 2n transfers. We have just shown that an+1 = 4an - 2N. Put cn = an - 2N/3, then cn+1 = 4cn.
We have c0 = a - 2N/3, so cn = 4na - 4n2N/3. Hence an = 4na - 4n2N/3 + 2N/3 = (4n + 2)a/3 -
2(4n - 1)b/3. We require an = b and hence a/b = (2.4n + 1)/(4n + 2).

Note that we have not so far proved that there is a solution for n. We have just showed that if
there is a solution then a/b has that ratio. It might be that one person has a negative number of
marbles at some point. Fortunately, we do not have to worry whether the question requires us
to show that n is possible, because it is easy to show that it is. The key observation is that if
one person goes negative, then they remain negative. But after 2n moves neither A nor B is
negative, so they cannot have been negative at any intermediate stage.

Problem 2

Find all integer solutions to m2n + 1 = m2 + 2mn + 2m + n.

Solution

Answer: (m, n) = (-1, -1), (0, 1), (1, -1), (2, -7), or (3, 7).

We have m2(n - 1) = 2m(n + 1) + n - 1 = 2m(n - 1) + 4m + (n - 1). Put k = n - 1 and this


becomes m2k = 2mk + 4m + k (*).

If k = 0, then m = 0. This gives the solution m = 0, n = 1. If m = 0, then k = 0, so we assume


m and k are non-zero. From (*) m must divide k and k must divide 4m. Put k = mh. Then h
divides 4, so h = ±1, ±2 or ±4. Also we can write (*) as (m2 - 2m - 1)h - 4 = 0.

If h = 1, then m2 - 2m - 5 = 0, which has no integral solutions. If h = -1, then m2 - 2m + 3 = 0,


which has no integral solutions. If h = 2, then m2 - 2m - 3 = 0, which has solutions m = -1 or
3. If h = -2, then m2 - 2m + 1 = 0, so m = 1. If h = 4, then m2 - 2m - 2 = 0, which has no
integral solutions. If h = -4, then m2 - 2m = 0, which has solutions m = 0, 2.

Problem 3
ABC is a triangle with AB greater than AC. AD is the angle bisector. E is the point on AB
such that ED is perpendicular to BC. F is the point on AC such that DE bisects ∠BEF. Show
that ∠FDC = ∠BAD.

Solution

CD bisects ∠A and DE is the external bisector of ∠FEA. Hence D is an excenter of AEF and
DF is an external bisector of ∠AFE.

∠BED = 90o - ∠B, so ∠AEF = 2 ∠B, so ∠AFE = ∠C - ∠B. Hence ∠CFD = (180o - (∠C -
∠B))/2 = 90o - ∠C/2 + ∠B/2. Hence ∠FDC = 180o - (90o - ∠C/2 + ∠B/2) - C = 90o - ∠B/2 -
∠C/2 = ∠A/2 = ∠BAD.

Problem 4

n dwarfs with heights 1, 2, 3, ... , n stand in a circle. S is the sum of the (non-negative)
differences between each adjacent pair of dwarfs. What are the maximum and minimum
possible values of S?

Solution

Answer: min 2n - 2, max [n2/2].

The minimum is obviously 2(n-1). The difference between 1 and n is n-1 and the sum of the
signed differences as we go from n to 1 either way round the circle must be n-1. The sum of
the unsigned differences must be at least as large, so S ≥ 2(n-1). This is achieved by the order
1, 2, 3, ... , n.

The maximum is almost obvious. If the numbers are a1, a2, ... , an. Then each difference is ai+1 -
ai or - ai+1 + ai. So the total of all the differences is k1a1 + k2a2 + ... + knan, where each ki is -2, 0
or 2 and their sum is 0. So permuting back to 1, 2, ... , n the sum is h1 + 2h2 + 3h3 + ... + nhn,
where each hi is 0 or ±2 and the sum of the hi is 0. If n = 2m, this is maximised by taking h1 =
h2 = ... = hm = -2 and hm+1 = ... = h2m = 2, giving a sum of 2m2. If n = 2m+1, it is maximised by
taking h1 = ... = hm = -2, hm+1 = 0, hm+2 = ... = h2m+1 = 2, giving a sum of 2m(m+1).

Checking that this can be achieved, we take: 1, 2m, 2, 2m-1, 3, 2m-2, ... , m-1, m+2, m, m+1
in the even case and 1, 2m+1, 2, 2m-1, 3, 2m-3, ... , m-1, m+3, m, m+2, m+1 in the odd case.
38th BMO 2002

Problem 1

From the foot of an altitude in an acute-angled triangle perpendiculars are drawn to the other
two sides. Show that the distance between their feet is independent of the choice of altitude.

Solution

Let the triangle be ABC and the altitude AD. Let M be the midpoint of AD, and E, F the feet
of the perpendiculars from D to AB and AC respectively. Angles AED and AFD are 90 deg,
so AEDF is cyclic and M is the center of the circle. Hence angle EMF = 2 angle A and EF = 2
ME sin A = AD sin A = 2 (area ABC) (sin A)/BC. Similarly the distances obtained starting
with the other two altitudes are 2 (area ABC) (sin B)/AC and 2 (area ABC)(sin C)/AB. But
these are all equal by the sine rule.

Problem 2

n people wish to sit at a round table which has n chairs. The first person takes an seat. The
second person sits one place to the right of the first person, the third person sits two places to
the right of the second person, the fourth person sits three places to the right of the third
person and so on. For which n is this possible?

Solution

Answer: n = 2m for m a non-negative integer.

Label the seats 0, 1, 2, ... , n-1. The kth person sits in seat k(k - 1)/2 for k = 1, 2, ... , n. So the
seating plan is possible iff the n numbers k(k - 1)/2 are incongruent mod n for k = 1, 2, ... , n.
Suppose first that n = 2m. We have h(h - 1)/2 - k(k - 1)/2 = (h - k)(h + k - 1)/2. If h and k are
unequal, then h - k and (h + k - 1) are non-zero. But they have opposite parity, and both are
less than 2n, so one is odd and the other is not divisible by a higher power of 2 than 2m. Thus
(h - k)(h + k - 1)/2 cannot be divisible by 2m. So the seating plan works for n a power of 2.

Conversely, suppose n = 2m(2a + 1). If a <= 2m, take h = 2m + a + 1, k = 2m - a. Then h - k = 2a


+ 1, h + k - 1 = 2m+1, so h(h - 1)/2 and k(k - 1)/2 are equal mod n. If a > 2m, then take h = 2m +
a + 1, k = a - 2m + 1. Then h - k = 2m+1, h + k - 1 = 2a + 1. So again h(h - 1)/2 and k(k - 1)/2 are
equal mod n. So no seating plan is possible if n is not a power of 2.
Problem 3

The real sequence x1, x1, x2, ... is defined by x0 = 1, xn+1 = (3xn + √(5xn2 - 4) )/2. Show that all
the terms are integers.

Solution

(2xn+1 - 3xn)2 = 5xn2 - 4, so xn+12 - 3xn+1xn + xn2 = -1. Subtracting the corresponding equation for
n-1 gives: xn+12 - 3xn(xn+1 - xn-1) - xn-12 = 0. So (xn+1 - xn)(xn+1 - 3xn + xn-1) = 0. But it is obvious
from the definition that xn+1 >= 3/2 xn, so xn+1 - xn is non-zero. Hence xn+1 = 3xn - xn-1. But x0 =
1, x1 = 2, so by a trivial induction all xn are integral.

Problem 4

S1, S2, ... , Sn are spheres of radius 1 arranged so that each touches exactly two others. P is a
point outside all the spheres. Let x1, x2, ... , xn be the distances from P to the n points of
contact between two spheres and y1, y2, ... , yn be the lengths of the tangents from P to the
spheres. Show that x1x2 ... xn ≥ y1y2 ... yn.

Solution

It is sufficient to show that if two circles centers O and O' both with radius 1 touch at X and P
is a point outside the circles with tangents PY, PZ to the two circles, then PY·PZ ≤ PX2. Let
the angle between PX and the line of centers be θ and the distances PX, PY, PZ be x, y, z
respectively. By the cosine formula, PO2 = x2 + 1 - 2x cos θ, so y2 = x2 - 2x cos θ. Similarly, z2
= x2 + 2x cos θ. Hence y2z2 = x4 - 4x2cos2θ ≤ x4.

You might also like